Sie sind auf Seite 1von 105

Universidad La Salle. Facultad Mexicana de Medicina.

Curso de Extensin Universitaria para la Preparacin del Examen Nacional para Aspirantes a Residencias Mdicas. Examen Mdulo de Gineco-obstetricia. 24 DE FEBRERO 2010. Modalidad a Distancia.

Nombre: Examen Mdulo I Ginecologa y Obstetricia. Duracin: 2 horas. Nmero de intentos: 3. Vigencia: 24 de Febrero del 2010. Horario: 7:00p.m. a 9:00 p.m. Programar aleatoriamente.

1.- Femenino de 36 aos nulpara tras 2 aos de relaciones sexuales sin contracepcin, que desde hace 1 ao presenta dismenorrea, dispareunia y sangrado vaginal intermenstrual. Cul sera la primera orientacin diagnstica?:

a) Insuficiencia lutenica. b) Enfermedad inflamatoria plvica. c) Dismenorrea funcional. d) Endometriosis.

La endometriosis consiste en la aparicin y crecimiento de tejido endometrial fuera del tero, sobre todo en la cavidad plvica como en los ovarios, detrs del tero, en los ligamentos uterinos, en la vejiga urinaria o en el intestino. Es menos frecuente que la endometriosis aparezca fuera del abdomen como en los pulmones o en otras partes del cuerpo. La endometriosis es una enfermedad relativamente frecuente, que puede afectar a cualquier mujer en edad frtil, desde la menarquia hasta la menopausia, aunque algunas veces, la endometriosis puede durar hasta despus de la menopausia. La endometriosis altera la calidad de vida de las mujeres que la padecen, afectando a sus relaciones de pareja, familiares, laborales y de reproduccin.

Sntomas Los sntomas clsicos son la dismenorrea, dolor plvico, dispareunia, sangrados intermestruales y en muchos casos, esterilidad. El dolor no tiene que ver con el tamao y la severidad de la lesin; generalmente cuanto menor es la lesin mayor dolor produce. El dolor se agrava con las menstruaciones y en los casos en que la lesin ocupa el fondo de saco de Douglas, puede dar dispareunia. Existe un aumento de la PGF2 alfa y PGE2 y un aumento de las contracciones uterinas que podra deberse a un depsito de endometrio en la cavidad peritoneal. La esterilidad debido a la endometriosis podra deberse a distintas causas de acuerdo a la severidad de la patologa. En los casos de endometriosis severa puede haber un factor tuboperitoneal con adherencias y alteracin en la anatoma de la pelvis que interfiera con el transporte del esperma y el vulo. En los casos de endometriosis leve hay varios mecanismos propuestos que justifican su relacin con la infertilidad: foliculognesis alterada, fase ltea inadecuada, fagocitosis espermtica, mala calidad ovocitaria, embriotoxicidad y alteracin a nivel de la implantacin. La produccin de prostaglandinas por el endometrio ectpico puede afectar la motilidad tubaria, la foliculognesis y la funcin del cuerpo lteo. Puede haber un aumento de la activacin de los macrfagos peritoneales en la endometriosis que cause la fagocitosis de los espermas o la secrecin de citoquinas que pueden ser txicas para el embrin. Segn algunos investigadores habra un 60% de las mujeres con endometriosis que presentan un sndrome de Folculo Luteinizado no roto (LUF) en el cual el folculo no se rompe en la ovulacin y el vulo queda atrapado.

Referencias bibliogrficas

1. Ruiz V. Endometriosis y fertilidad. Ed. Acosta y Warman, pp. 99 2. Lpes,VH. Palomo E. Incidencias de endometriosis en una poblacin infrtil. XXI Congreso nacional de Ginecologa y Obtetricia. Guatemala, 1993. 3. El-Eoley, et al. Danazol but not ginadotropin releasing hormone agonists suppresses autoantibodies in endomeriosis. Fertil Steril 1990; 54:725 4. Acosta AA. Buttram VC Jr. Besch PK, Malinak LR, Van Der Heyden J. A.proposed classfication of pelvic endometriosis. Obstet Gynecol 1973;42:19. 5. Buttran VC Jr. Evolution of the revised American Fertility classification of endometriosis. Fert. Steril 1985; 43: 347 6. Lpez VH. Tratamiento mdico-quirrgico de la endometriosis. Simposio El rostro cambiante de la endometriosis panam 3. 12. 1993.

7. Steinleitner A. Heterolous transplation of activated murine peritonel macrophages inhibitis gamete interaction in vivo; A paradigm fo endometriosis associted subfertility. Fertil Steril 1990; 54:725.
2.- Femenino de 32 aos segunda gesta a trmino sin anormalidades en el transcurso de ste. Inicia trabajo de parto de forma espontnea, con evolucin normal hasta que se rompe la bolsa, con una dilatacin de 4 cm. A partir de entonces, comienza con hemorragia de sangre roja, en moderada cantidad y aparecen signos de sufrimiento fetal agudo. El estado general de la mujer es bueno y la dinmica uterina es normal. Este cuadro corresponde a: a) Placenta previa central. b) Abruptio placentae. c) Rotura uterina. d) Rotura de vasa previa.

La vasa previa es una condicin de alto riesgo obsttrico en la cual vasos fetales o placentarios cruzan el segmento uterino por debajo de presentacin. Estos vasos estn desprotegidos de gelatina de Wharton o tejido placentario, lo que los hace altamente vulnerables y susceptibles de ruptura o laceraciones en cualquier perodo del embarazo, principalmente en el momento del parto. Tambin es frecuente la compresin de estos vasos, especialmente durante el tercer trimestre de la gestacin, lo que puede condicionar asfixia y muerte fetal.

Esta condicin ocurre como resultado de que vasos velamentosos cruzan por el segmento uterino debido a una insercin velamentosa del cordn, situacin en la cual el cordn umbilical se inserta en las membranas ovulares en vez del tejido placentario (vasa previa tipo I), o por el cruce de vasos fetalesentre uno o ms lbulos accesorios de la placenta (vasa previa tipo II) (Figura 1).

Figura 1. A, Vasa previa tipo I, debida a insercin velamentosa. B, Vasa previa tipo II, debida a cotiledones aberrrantes. Reproducido de Daly-Jones y cols. Ultrasound 2008.

Vasa previa se puede presentar si existe alguna (o ninguna) de las siguientes condiciones: placenta baja (que puede ser causa de abortos previos seguidos por legrado o por operaciones uterinas, que provocan cicatrices en el tero), placenta bilobada o de lbulo succensuriado, embarazos resultado de fertilizacin in vitro, o embarazos mltiples (5-6). El sangrado por vasa previa no es doloroso. Otros sangrados por complicaciones o por nacimiento no necesariamente son sin dolor.

Referencias

1. Oyalese Y, Smulian JC. Placenta previa, placenta acreta, and vasa previa. Obstet Gynecol 2006; 107: 927-941. 2. Oyalesse KO, Turner M, Less C, Campbell S. Vasa previa: an avoidable obstetric tragedy. Obstet Gynecol Surv 1999; 54: 138-145. 3. Seplveda W, Sebire NJ, Harris R, Nyberg DA. The placenta, umbilical cord, and membranas. In Diagnostic Imaging of Fetal Anomalies, Nyberg DA, MaGahan JP, Pretorius DH, Pilu G (eds). Philadelphia, PA: Lippicont Williams & Wilkins 2003, 85-132. 4. Daly-Jones E, John A, Leahy A, McKenna C, Sepulveda W. Vasa praevia; a preventable tragedy. Ultrasound 2006; 16: 8-14. 5. Derbala Y, Grochal F, Jeanty P. Vasa previa. J Prenat Med 2007; 1: 2-13. 6. Fung TY, Lau TK. Poor perinatal outcome associated with vasa previa. It is preventable? A report of three cases and review of the literature. Ultrasound Obstet Gynecol 1998; 12: 430-433. 7. Robert JA, Sepulveda W. Fetal exsanguination from ruptured vasa previa: still a catastrophic event in modern obstetrics. J Obstet Gynaecol 2003; 23: 574. 8. Cordero DR, Helfgott AW, Landy HJ, et al. A non-hemorrhagic manifestation of vasa previa: a clinicopathologic case report. Obstet Gynecol 1993; 82: 698-700. 9. Schachter M, Tovbin Y, Arieli S, et al. In vitro fertilization as a risk factor for vasa previa. Fertil Steril 2002; 78: 642-643.

3.-Recibe usted los resultados histopatolgicos de una paciente de 24 aos de edad que acudi a revisin rutinaria, los resultados reportan imagen histolgica de coilocitos lo cual sugiere infeccin por:

a) b) c) d)

Herpes virus tipo 2. Citomegalovirus. Virus del papiloma humano. Vaginosis bacteriana.

El coilocito es un tipo de clula hallada en lesiones precancerosas cervicales. Tambin es comn apreciarla microscpicamente en lesiones reaccionales en la mucosa oral, debido a su similitud con la mucosa vaginal, en enfermedades como Papilomas, o en Condiloma acuminado. El coilocito es la manifestacin clsica de la infeccin por VPH en la clula. Fue descrito por primera vez por Koss y Durfee en 1956. Esta clula tambin ha sido llamada clula en baln. El coilocito es una clula epitelial escamosa, ms comnmente superficial e intermedia, aunque tambin puede verse en clulas parabasales y metaplsicas. Esta clula presenta cambios tpicos tanto en su ncleo como en su citoplasma, pierde los bordes angulados usuales de la clula escamosa superficial y su forma tiende a ser redondeada y ovoide. El citoplasma muestra una condensacin perifrica que le da un aspecto en asa de alambre, es opaco, denso y de aspecto creo, anfoflico, acidoflico o de color rojo/naranja brillante. Adems se observa una gran cavidad o halo con un margen muy bien definido, de forma oval o ligeramente festoneado. El ncleo de la clula se localiza de manera excntrica, lo que lo convierte en un halo paranuclear, no perinuclear. Ocasionalmente puede encontrarse material fagocitado dentro del espacio coiloctico.

Lesin Intraepitelial de Bajo Grado. Alteraciones Celulares compatibles con Infeccin (Coilocitos).

Referencia: Schlecht, N.F., Kulaga, S., Robitaille, J., Ferreira, S., Santos, M., Miyamura, R.A., Duarte-Franco, E., Rohan, T.E., Ferenczy, A., Villa, L.L., & Franco, E.L. (2002) Persistent Human Papillomavirus Infection as a Predictor of Cervical Intraepithelial Neoplasia. JAMA, 286, 3106-3114.

4.- Paciente de 26 aos con deseo de un embarazo, antecedentes de G3 A2 - P1 se le realiza una histerosalpingografa, se constata que existe un sndrome de Asherman. Ello significa que se trata de:

a) b) c) d)

Sinequias uterinas. tero bicorne. Endometriosis en la trompa. Insuficiencia istmico cervical.

El sndrome de Asherman es una enfermedad ginecolgica rara que se caracteriza por la presencia de sinequias (adherencias) intrauterinas que pueden ocasionar amenorrea (ausencia de perodos menstruales regulares) e infertilidad.

En 1894 Heinrich Fritsch describe por primera vez la presencia de sinequias intrauterinas de tipo postraumtico, en una paciente que desarroll una amenorrea secundaria a un curetaje. Posteriormente en 1927 Bass inform de veinte casos de atresia (oclusin de una abertura natural) cervical tras abortos inducidos, pero no fue hasta 1948, cuando Joseph G. Asherman recopil la informacin hasta entonces existente y acu el nombre con el que se conoce actualmente a la enfermedad.

Asherman describi originalmente dos tipos diferentes de amenorrea secundaria, en funcin de su etiologa (estudio de las causas de las enfermedades): la amenorrea traumtica atrtica, debida a estenosis del orificio cervical interno y la amenorrea debida a adherencias intrauterinas. Posteriormente ambas entidades se agruparon en una nica entidad bajo el nombre de sndrome de Asherman.

Suele presentarse en mayor proporcin tras dilataciones y curetajes uterinos de repeticin y sobre todo si se realizan durante el embarazo o si existe infeccin uterina en el momento en el que se realizan estas intervenciones.

Las adherencias intrauterinas pueden producirse debido a cualquier factor que lleve a una destruccin de las paredes del miometrio (capa muscular de la pared del tero). Sin embargo, hay que distinguir entre factores predisponentes, siendo el principal de ellos el embarazo y factores causales, entre los que se encuentran: traumatismos uterinos, intervenciones quirrgicas que afecten al tero, agentes fsicos o qumicos e infecciones uterinas por tuberculosis o esquistosomiasis. En cualquier caso, el factor ms importante es el trauma uterino en el momento del parto o el puerperio.

El cuadro clnico es muy variable y las manifestaciones clnicas varan con el grado de oclusin de la cavidad uterina y la severidad de las adherencias, pudiendo presentarse: esterilidad cuando la oclusin de la cavidad uterina incluye porciones proximales (ms cerca de un centro, tronco o lnea media) de las trompas de Falopio o cuando las adherencias impiden la nidacin del huevo; las pacientes presentan con frecuencia amenorrea, oligomenorrea (disminucin de la frecuencia de las menstruaciones), dismenorrea (menstruacin dolorosa) y abortos repetidos. Referencia: 1. Hysteroscopic treatment of severe Asherman's syndrome and subsequent fertility. Capella-Allouc S; Hum Reprod, 1999 May.

5.- Uno de los siguientes cambios fisiopatolgicos tienen lugar durante el embarazo normal:

a) Hiperuricemia. b) Proteinuria. c) Hipertensin. d) Un incremento de ms del 40% en la filtracin glomerular.

Allen R. M. MMS Medicina Interna. 5. Edicin. National Medical Series. Mc. Graw Hill. 2006. (captulo 6, parte I: XV A 1, 2). El ndice de filtracin glomerular (GFR) se incrementa alrededor de 40% en el embarazo. De hecho hay un descenso de la concentracin srica de cido rico y un aumento de la depuracin de cido rico. No se ve proteinuria en el embarazo y el dato de una mayor excrecin urinaria de protenas sugiere la presencia de nefropata o preeclampsia subyacente. La presin arterial disminuye durante el embarazo; por tanto, cualquier grado de aumento representa hipertensin importante. Por ltimo, a menudo hay alcalosis respiratoria y no metablica y esto provoca una baja de la concentracin srica de bicarbonato.

6.- Se presenta a consulta paciente de 25 aos de edad refiere que desde hace varios das ha presentado flujo vaginal cuyas caractersticas son: blanquecino, grumoso, sin mal olor, muy irritante El agente causal ms probable en esta patologa es? a) Gardnerella b) Tricomonas c) Cndida albicans d) Gonococo

Leucorreas micticas: los hongos dan un flujo abundante, blanco, con grumos (aspecto como de quesillo cortado) sin mal olor y que es muy irritante de la piel de la regin genital y por lo tanto genera gran ardor vaginal y prurito (picazn). Son muy frecuentes y por lo general son producidas por el hongo Candida albicans. Tabla I. Caractersticas del flujo vaginal segn la causa Cantidad Candidiasis Escasamoderada Color Blancoamarillento Amarilloverdoso Blancogrisceo Consistencia Grumosa Espumosa Homogneoadherente Olor Indiferente Maloliente Maloliente

Tricomonas Aumentada Vaginosis Moderada

Referencia:

Speroff Leon and Fritz Marc A. Clinical Gynecologic endocrinology and infertility. 7 ed. Philadelphia: Lippincott Williams and Wilkins, 2005. p. 25 44.

7.- Paciente de 50 aos, con mioma uterino de tamao equivalente a una gestacin de 12 semanas, que presenta hipermenorreas y hemoglobinemia de 9 gr%. No se demuestra patologa asociada. Se encuentra en espera para la prctica de una histerectoma programada a realizar en 4 meses. En esta paciente est indicado el tratamiento preoperatorio con: a) Estrgenos. b) Inhibidores de la fibrinlisis. c) Anlogos de la GnRH. d) Derivados del cornezuelo del centeno.

Anlogos de la GnRH: Son derivados de la hormona GnRH en donde se ha realizado una sustitucin peptdica en posicin 6 y en algunos casos en la 10, obteniendo compuestos hasta unas 200 veces ms potentes debido a mayor afinidad por los receptores y a su resistencia a la degradacin por peptidasas. Aunque su accin inicial produce un incremento en la produccin de FSH-LH (efecto flure-up o llamarada) tras 56 das de exposicin contnua, los receptores son internalizados producindose un estado de hipogonadismo hipogonadotropo y niveles de estradiol similares a los de la postmenopausia. Los anlogos de GnRH estn disponibles en distintas frmulas: administracin nasal (varias aplicaciones al da), subcutnea (aplicacin diaria) o intramuscular (preparados depot mensuales o trimestrales) (Shaw RW 1999). Marco Filicori y sus colaboradores de la Universidad de Bolonia fueron los primeros en utilizar en 1983 los aGnRH en un estudio que confirm su eficacia para reducir el tamao de los miomas uterinos y secundariamente sntomas como alteraciones menstruales, dolor plvico y sntomas de presin local. Otros autores como Minaguchi H y colaboradores continan comprobando la efectividad de los anlogos de la GnRH en el tratamiento del mioma uterino tras evaluar en el ao 2000 seis estudios con un total de 602 pacientes tratadas con nafarelina. La disminucin del tamao se calcula entre un 30-70%, y se ha observado como el mayor porcentaje de reduccin ocurre tras el primer mes de tratamiento, no existiendo reducciones o siendo stas mnimas despus del tercer mes (Healy et al 1986; Friedman et al 1989; Matta et al 1989; Williams y Shaw 1990). En miomas pediculados o con gran proporcin de calcio o colgeno (hialinizacin) la repuesta es tambin menor. Debe tenerse en cuenta que si despus de dos meses de tratamiento no se ha producido un significativo descenso del tamao del mioma, ste ya no debe ser esperado y debe pensarse en la posibilidad de la existencia de un tumor muscular maligno no diagnosticado (Messia AF et al 1998). Tras finalizar el tratamiento y recuperarse el estado de hipogonadismo, el mioma retorna rpidamente a su tamao inicial (Friedman AJ et al 1987; Matta WH et al 1989).

En casos prximos a la menopausia, la reduccin del tamao del mioma y su sintomatologa, permitira hablar de una solucin mdica del problema, pero en todo caso la utilizacin de aGnRH facilitara la intervencin quirrgica al acortar el tiempo de intervencin, la hemorragia y el acceso a localizaciones complicadas como el caso de miomas interligamentarios o situados en istmo o crvix. En el caso de la ciruga histeroscpica la reduccin del tiempo de ciruga permitira reducir el volumen de fluidos aportado a cavidad uterina y los riegos de absorcin e hiponatremia. Los mecanismos de accin por los que los aGnRH actan son: o Hipoestrogenemia: es necesario mantener la hipoestrogenemia, pues la elevacin de sus niveles lleva a un rpido incremento del tamao del mioma. El crecimiento del mioma es dependiente de los niveles de estrgenos (aumentan de tamao con el embarazo y se reducen durante la menopausia o el tratamiento con aGnRH, pudiendo volver a crecer durante la THS), pero aunque los estrgenos parecen ser importantes en el crecimiento del mioma, su relacin debe ser algo ms compleja pues no se han descrito incrementos significativos del tamao de miomas durante el tratamiento con gonadotrofinas en RA (situaciones con elevados niveles de E2), algunos de ellos no se modifican durante el embarazo o incluso decrecen y se han encontrado crecimientos despus del tratamiento con citrato de clomifeno (antiestrgeno).

En relacin con la hipoestrogenemia podran estar los cambios inducidos en el flujo vascular uterino (incrementos en el ndice de resistencia de las arterias uterinas) que suponen una reduccin de la vascularizacin o las modificaciones de distintos factores de crecimiento. Cambios histolgicos: El tratamiento con aGnRH puede producir degeneracin roja, infiltracin linfocitaria, y necrosis, as como reduccin de la proliferacin celular e incremento de la apoptosis. Pero en otras circunstancias no es posible encontrar 7 diferencias. No se ha encontrado una relacin entre los cambios histolgicos y el porcentaje de reduccin del tamao del tero, y existe una gran variabilidad entre distintas pacientes o entre distintos miomas de una misma paciente, no existiendo pues un patrn histolgico caracterstico de respuesta ante el tratamiento con aGnRH.

BIBLIOGRAFA 1. Abad L, Abad de Velasco L, Parilla JJ. Etiopatogenia. Papel de las hormonas esteroideas, factores de crecimiento y otras sustancias. Cuad Med Reprod 1999;5(1):15-29. 2. Albano C, Platteau P, Devroey P. Gonadotropin-releasing hormone antagonist: how good is the new hope? Curr Opin Obstet Gynecol 2001;13(3):257-62.

3. Coutinho EM.Treatment of large fibroids with high doses of gestrinone. Gynecol Obstet Invest 1990;30(1):44-47. 4. Chavez NF, Stewart EA. Medical treatment of uterine fibroids. Clin Obstet Gynecol 2001;44(2):327-84. 5. De Leo V, la Marca A, Morgante G. Shortterm treatment of uterine fibromyomas with danazol. Gynecol Obstet Invest 1999;47(4):258-262. 6. Eldar-Geva T, Healy DL. Other medical management of uterine fibroids. Baillieres Clin Obstet Gynaecol 1998;12(2):269-88. 7. Felberbaum RE, Germer U, Ludwig M, Riethmuller-Winzen H, Heise S, Buttge I, Bauer O, Reissmann T, Engel J, Diedrich K. Treatment of uterine fibroids with a slow-release formulation of the gonadotrophin releasing hormone antagonist Cetrorelix.HumReprod 1998;13(6):1660-8.

8.- Se trata de paciente de 19 aos G1 P1, acude al servicio mdico por referir, una secrecin transvaginal bastante lquida, de baja viscosidad, maloliente de color amarillo y gris, espumoso. Esta entidad es propia de infeccin por: a) Cndida albicans. b) Gardenerella. c) Tricomonas. d) Gonococos.

Es la presencia de protozoos flagelados, pertenecientes a la Familia Tricomonadidae y al gnero tricomonas, que parasitan el tracto reproductor del hombre. En el ser humano ocurre la infeccin por la especie Tricomonas vaginalis que se encuentra en el tracto genitourinario de la mujer y del hombre, en todo el mundo, este parsito fue descrito por primera vez en 1836 por el Francs Donn, habita en la vagina y uretra de la mujer y en la glndula prosttica, vesculas seminales y uretra del hombre. A diferencia de las infecciones por hongos la tricomoniasis se transmite a travs de las relaciones sexuales, considerndose por tanto una infeccin de transmisin sexual (ITS).

MANIFESTACIONES CLINICAS: Los sntomas se desarrollan de 4 a 30 das despus de la infeccin; periodos de incubacin ms cortos se relacionan con una enfermedad ms grave. La presencia de sntomas es ms frecuente e importante en el sexo femenino. En el hombre provoca escasos o nulos sntomas. Las variadas formas clnicas de la enfermedad dependen probablemente del nmero y virulencia del parsito y de la resistencia del hospedero. Se plantea que los sntomas son mayor cuanto mayor es el nmero de parsitos y cuanto ms alcalino sea el Ph. vaginal. La importancia de conocer el grupo de individuos asintomticos, radica en que ellos actan como portadores sanos y pueden transmitir sexualmente el parsito a otras personas, Botero & Restrepo (1992), son de la opinin de la infeccin se puede presentar en tres formas: vaginitis asintomtica, vulvo vaginitis aguda o crnica y uretritis. La manifestacin clnica mas frecuente es la vulvo vaginitis de evolucin aguda o crnica y el signo o sntoma mas frecuente es la leucorrea, que se presenta como un flujo de tipo purulento y espumoso. La leucorrea puede ser variable en cantidad de color amarillento verdoso o gris, inodora o de olor fuerte. Otros sntomas son el prurito vulvar, ardor y la irritacin genital dolorosa que puede llegar a provocar intensa dispareunia.

Referencia: Krieger JN and Alderete JF. Trichomonas vaginalis and trichomoniasis. In: K. Holmes, P. Markh, P. Sparling et al (eds). Sexually Transmitted Diseases, 3rd Edition. New York: McGraw-Hill, 1999, 587-604.

9.- Seale cul de los tumores siguientes es el ms frecuente en el tracto genital femenino: a) Tumor de los senos endodrmicos. b) Carcinoma seroso-papilar de superficie ovrica. c) Leiomiona uterino. d) Adenocarcinoma mucosecretor de crvix uterino.

LEIOMIOMA UTERINO O FIBROMIOMA UTERINO. Lesin tumoral generalmente benigna ms frecuente en la mujer durante la etapa reproductiva. Su origen histolgico muscular tiende a ser mltiples, de localizacin y tamao variable. EPIDEMIOLOGIA. Refiere la literatura que entre el 4 al 30% de las pacientes entre la 3 y 5 dcada de la Vida presentan leiomiomatosis uterina, siendo de mayor frecuencia en la raza negra. FISIOPATOLOGA De tipo multifactorial, se han sugerido factores hormonales, por ser mas frecuente en la etapa reproductiva. As como el uso de anticonceptivos orales como factor causal; la infeccin por virus de papiloma humano, por la induccin de tumores conjuntos en animales. Se ha cuestionado a la miomatosis con la hormona del crecimiento. La estimulacin estrognica sostenida que provoca la induccin de clulas semejantes a los fibroblastos a partir de clulas totipotenciales con estimulacin progestacional se diferencian en clulas musculares lisas. CLASIFICACION. Miomatosis subserosa: situada por debajo del peritoneo visceral del tero. Miomatosis intramural: a partir de la pared muscular uterina. Miomatosis submucosos: se proyecta hacia la cavidad endometrial.; los cuales pueden protuir por el crvix y son conocidos como miomas abortivos pediculados. CUADRO CLINICO. En su mayora son asintomticos, el dolor se presenta generalmente por degeneracin o torsin del mioma. El dolor tipo clico puede presentarse de leve a moderado tiende aumentar durante el periodo menstrual. En ocasiones se presenta como pesantez en bajo vientre de predominio vespertino qua aumenta con el coito. La hemorragia uterina anormal es de tipo hiperproiopolimenorrea en ms de la mitad de los casos, manifestaciones de un sndrome anmico que se corrobora con exmenes de laboratorio. S se presenta aumento de volumen abdominal debe descartarse embarazo.

BIBLIOGRAFA. 1. Benson . Manual de obstetricia y Ginecologa. 7 edicin, Mc Graw Hill. 2. Botella Llusi J. Tratado de Ginecologa. 14 Edicin, ed. Diaz de Santos. 3. Danforth, Tratado de Obstetricia y Ginecologa, 7 edicin, ed. Mc Graw Hill 4. Enciclopedia Medico Quirrgica, Ginecologa Obstetricia. tomo 5. Editorial Scientifiques et Medicales Elsevier 1999. 5. INPER. Normas y procedimientos de obstetricia y ginecologa, 1998. 6. Llaca y Fernndez Alba. Obstetricia Clnica. Editorial Mc Graw Hill.

10.-Recibe un frotis vaginal el cual reporta la presencia al microscopio de clulas clave las cuales son propias de infeccin por: a) Cndida albicans. b) Gardnerella. c) Tricomonas. d) Gonococos.

La infeccin por gardnerella (bacteria) tiende a producir una secrecin blanca, gris o de color amarillo turbio, con un olor ftido o a "pescado" que aumenta cuando la secrecin se vuelve alcalina, como sucede despus del coito o de lavarse con jabn. Puede haber prurito o irritacin vulvar, pero por lo general no son muy pronunciados. Al microscopio se aprecia las famosas celulas clave. Realizando el diagnostico diferencial.

Clulas escamosas de capa intermedia alta, algunas de ellas con ncleos picnticos, y dos de ellas con el citoplasma cubierto por formas cocceas que borran sus bordes, dndole el aspecto de clula rebozada "clula clave". Gardnerella. .

Referencia: Pernoll M. Enfermedades de trasmisin sexual. En: Manual de Obstetricia y Ginecologa. Benson/Pernoll Editores.Editorial Interamericana S A. Mxico DF, 1994

11.- Al realizar el diagnstico de hirsutismo asociado a ovario poliqustico, usted elige el siguiente frmaco para su tratamiento por ser el ms adecuado:

a) Clomifeno. b) Estrgenos. c) Corticoide. d) Acetato de ciproterona.

El acetato de ciproterona parece ser ms efectivo que otros frmacos para el hirsutismo en mujeres causado por la produccin ovrica excesiva de andrgenos Una de las causas de hirsutismo (crecimiento piloso excesivo) en mujeres es la hiperproduccin de andrgenos a partir del ovario. Varios frmacos pueden utilizarse para contrarrestar los efectos del andrgeno. El acetato de ciproterona es un frmaco antiandrognico. Los efectos adversos informados con su uso fueron aumento de peso, depresin, fatiga, sntomas mamarios y disfuncin sexual. La revisin de los ensayos encontr que el acetato de ciproterona parece ejercer un efecto en el hirsutismo similar a otros frmacos utilizados para el tratamiento del hirsutismo por exceso de andrgenos. No existen pruebas suficientes para comparar los efectos adversos de las opciones de tratamiento.

Referencia: Van der Spuy ZM, le Roux PA. Acetato de ciproterona para el hirsutismo (Revisin Cochrane traducida). En: La Biblioteca Cochrane Plus, nmero 4, 2007. Oxford, Update Software Ltd. Disponible en: http://www.update-software.com. (Traducida de The Cochrane Library, 2007 Issue 4. Chichester, UK: John Wiley & Sons, Ltd.).

12.- Es una indicacin absoluta para la realizacin de operacin cesrea? a) b) c) d) Placenta Previa central Total. Cardiopata. Presentacin plvica. Desproporcin cfaloplvica.

La indicacin para realizar una operacin cesrea puede ser por causas: maternas, fetales o mixtas. A.Causas maternas. Distocia de partes seas (desproporcin cefaloplvica) Estrechez plvica. Pelvis asimtrica o deformada. Tumores seos de la pelvis. Distocia de partes blandas Malformaciones congnitas. Tumores del cuerpo o segmento uterino, cerviz, vagina y vulva que obstruyen el conducto del parto. Ciruga previa del segmento y/o cuerpo uterino, incluyendo operaciones cesreas previas. Ciruga previa del cervix, vagina y vulva que interfiere con el progreso adecuado del trabajo de parto. Distocia de la contraccin: Hemorragia normoinserta) (placenta previa o desprendimiento prematuro de la placenta

B. Causas fetales: Macrosomia fetal que condiciona desproporcion cefalopelvica. Alteraciones de la situacin, presentacin o actitud fetal. Prolapso del cordon umbilical. Sufrimiento fetal. Malformaciones fetales incompatibles con el parto. Embarazo prolongado.

Referencia: 1. Adair, C.D., Snchez, R.L., Gaudier, F.l., Kaunitz, A.M., Mc Dyer, D.C. and Briones, D. Labor induction in patients with previous cesarean section. Am. J. Perinatol. 12:450-454, 1995.

INDICACIOINDICACI

S
13.- Mujer de 23 aos con ndulo mamario palpable de aparicin brusca. La ecografa revela un ndulo anecognico, de limites muy precisos, morfologa regular y refuerzo posterior, nico de 3.5 cms. de dimetro. Cul ser es el diagnstico ms probable?: a) Quiste. b) Displasia fibrosa. c) Fibroadenoma. d) Cncer.

Quistes. Los quistes mamarios son fciles de detectar con la ecosonografa. Pueden ser lesiones nicas o mltiples que se observan como imgenes redondeadas, anecognicas, de paredes delgadas, contornos bien definidos, con importante reforzamiento acstico posterior y sombras laterales delgadas. Pueden presentar septos intraqusticos y, en ocasiones, se pueden observar ecos internos que sugieren detritus celulares o proceso inflamatorio. Se debe descartar la presencia de lesiones intraqusticas o la coexistencia de otras alteraciones benignas o malignas. En caso de ser sintomticos, el tratamiento adecuado es la puncin y aspiracin de la lesin con aguja guiada por palpacin o ecosonografa de acuerdo con ell tamao, profundidad y caractersticas del contenido. El uso del ultrasonido garantiza el vaciamiento completo.

REFERENCIAS Barth V, Prechtel K. Mama normal. En: Barth V, Prechtel K, editores. Atlas de patologa de la glndula mamaria. 2da ed. Madrid: Editorial Mdica Panamericana, 1991. Bush H, McCredie A. Carcinoma of the breast during pregnancy and lactation. In: Allen HH, Nisker JA. Cancer in pregnancy. New York: Futura Publishing Co. Inc., 1986. Byrd BF, Bayer DS, Robertson JC, Stephenson JE Jr. Treatment of breast tumor associated with pregnancy and lactation. Ann Surg. 1962; 155:940-7.

14.- Femenino de 23 aos de edad, refiere que desde hace un par de meses ha presentado hemorragia irregular o postcoital usted debe sospechar en cervicitis por:

a) Chlamydia. b) Micoplasma. c) Gardnerella. d) Candida.

Segn los datos de la Organizacin Mundial de la Salud, anualmente se detectan 89 000 000 de nuevas infecciones por Chlamydia trachomatis en el mundo. Esta infeccin provoca uretritis y cervicitis, y las secuelas incluyen enfermedad inflamatoria plvica, embarazo ectpico, infertilidad por dao tubrico, epididimitis, proctitis y artritis reactiva. Se considera principalmente un problema de salud en la mujer, en ella las manifestaciones y consecuencias son ms dainas para la salud reproductiva. Los individuos infectados con Chlamydia trachomatis pueden portar el microorganismo por meses o aos y transmitir la enfermedad a sus parejas sexuales. Su diagnstico sigue siendo un reto, ya que quienes la padecen presentan sntomas muy leves o son portadores asintomticos.

Manifestaciones clnicas:

La cervicitis es la manifestacin clnica ms frecuente de la infeccin por C. trachomatis en la mujer. Sin embargo, el 70% de las mujeres infectadas no tienen sntomas, mientras que en el tercio restante las evidencias clnicas son poco especficas de infeccin, como flujo genital, dolor abdominal o pelviano, sangrado y/o disuria.

La presencia de disuria puede indicar una uretritis acompaante, lo que sucede en el 35% de los casos. En otras oportunidades, solo la uretra est comprometida, y la infeccin uretral se manifiesta como piuria o disuria con cultivo negativo (23% de los casos). El diagnstico se realiza al examinar el hisopado endocervical, que muestra flujo amarillento o verdoso con ms de 10 PMN por campo de inmersin en el examen de Gram. Este resultado define la cervicitis mucopurulenta (CMP) la cual tambin puede ser producida en casos de infeccin por gonococo o mixta (C. trachomatis y gonococo). Por lo tanto, el diagnstico debe confirmarse mediante estudios de mayor especificidad, como las tcnicas moleculares (test de ligasa, PCR), que tienen una sensibilidad del 96% aproximadamente, o la deteccin del antgeno por tcnica de ELISA, con una sensibilidad del 75%. Tambin se ha demostrado que las tcnicas moleculares en el primer chorro de

orina son especficas y altamente sensibles.

Cuatro de cada diez mujeres con cervicitis no tratada adquieren enfermedad inflamatoria pelviana (EPI), con mayor riesgo de sufrir embarazo ectpico, infertilidad y dolor crnico pelviano. El riesgo de infertilidad se eleva segn el nmero e intensidad de los episodios: alrededor del 10% despus de un episodio, del 30% despus de dos, y mayor del 50% si ha habido tres o ms episodios. Por otro lado, el embarazo ectpico es cinco a siete veces ms frecuente cuando se trata de pacientes con antecedentes de EPI.

Tratamiento

Azitromicina 1 g VO dosis nica Doxiciclina 100 mg VO cada 12 horas por 7 das Eritromicina 500 mg VO cada 6 horas por 7 das Ofloxacina 300 mg VO cada 12 horas por 7das Levofloxacina 500 mg VO cada 24 horas por 7 das Referencia: Basado en Guas Clnicas para el manejo de las ITS, OMS, 2003 / Norma Oficial Mexicana NOM -039-SSA2-2002

CLAMIDIA

TRACHOMATIS

Berek J. (2002) Ginecologa de NOVAK. Mxico. Ed. Mc Graw Hill Interamericana. Pag 293.

15.- En una paciente que cursa con 12 semanas de gestacin que lleva tres das sangrando por genitales, con nuseas continuas y constantes, tero mayor aumentado de tamao y valores de beta HCG muy elevados, debemos sospechar de:

a) Amenaza de aborto. b) Aborto diferido. c) Mola hidatdica. d) Amenaza de aborto en un tero con miomas.

La enfermedad trofoblstica gestacional agrupa a diferentes entidades interrelacionadas: mola completa, generalmente diploide con origen cromosmico paterno, mola parcial generalmente triploide, tumor trofoblstico del lecho placentario y coriocarcinoma, con tendencias variables a la invasin local y a las metstasis, cuyo denominador comn es la hipersecrecin de hCG. El coriocarcinoma es diploide y proviene de ambos progenitores, excluyendo probablemente su origen directo en la mola completa. El tumor trofoblstico del lecho placentario est constituido por trofoblasto mononuclear intermedio no conteniendo vellosidades corinicas e inmunohistoquimicamente caracterizado por expresar muchas de sus clulas hPL y unas pocas hCG.

Cuadro clnico. Tras un periodo de amenorrea, y a partir del segundo mes, hay un aumento de los sntomas subjetivos del embarazo, sobre todo nuseas y vmitos o hipermesis en un 30%. Hay metrorragias irregulares en el 96% de los casos, en principio de escasa cantidad, pero que se va incrementando. Dicha hemorragia no es continua sino que se repite cada dos o tres das, de color roja o negruzca, que se produce por la ruptura de vasos maternos al separarse las vesculas de la decidua. Ocasionalmente, hay expulsin de restos molares, que lo refiere la paciente como expulsin de vesculas en el 11 % de los casos y que es patognomnico pero aparece tardamente. Tambin puede presentar mal estado general, dolor difuso en hipogastrio y anemia. Esta ltima en relacin con las metrorragias. Pueden haber signos y sntomas de hipertiroidismo, como taquicardia, sudoracin, y temblores, en el 7%, y es debido a que la fraccin -hCG es similar a la hormona TSH. Existen signos de preeclampsia o hipertensin gestacional del primer trimestre hasta en el 50% de los casos, y signos de insuficiencia respiratoria aguda en el 2% por embolismo pulmonar de las clulas trofoblsticas, o por la asociacin entre hipertiroidismo e hipertensin arterial. Como complicaciones pueden aparecer coagulopatas y metstasis. A la exploracin el tero esta aumentado de tamao, en el 50% de los casos, por encima de lo que correspondera a la edad gestacional,3 de consistencia blanda, sin signos de actividad fetal a partir de la semana 12 y siempre que se trate de una mola total. Tambin puede que la paciente presente un tamao uterino menor al esperado para la edad gestacional.3 El cuello est cerrado, con metrorragia en cantidad variable, y raramente se observa la expulsin de vesculas. Pueden haber quistes teca-lutenicos bilaterales en ovario en cerca del 20% de los casos,3 debido al estmulo de la -hCG.

La enfermedad trofoblstica maligna va a cursar con metrorragias por lo general intensas, hay una elevacin de la -hCG y en la ecografa se objetiva la cavidad uterina con signos de ocupacin atpica.

Diagnstico Por la clnica, y pruebas complementarias como la determinacin de la -hCG y la ecografa. La determinacin de la -hCG se basa en que el trofoblasto produce la hormona gonadotropina corinica, presentando cifras elevadas, y su cuantificacin va a servir para diagnstico, valorar el pronstico, y el seguimiento postratamiento. La ecografa revela un tero aumentado de tamao que no corresponde con la amenorrea, con ecos en su interior, puntiformes que corresponderan a las vesculas y que asemejan copos de nieve o panal de abeja. No se aprecia saco gestacional ni estructuras fetales y, en ambos ovarios se aprecian quistes teca-lutenicos como formaciones ovricas redondas, econegativas, con mltiples tabiques en su interior.

BIBLIOGRAFA 1. Mazur MT, Kurman RJ. Gestational trophoblastic disease and related lesions. En: Kurman RJ editor. Blaunsteins pathology of the female genital tract. 4th ed. New York: Springer-Verlag. 1994, p. 1049-93. Kurman RJ, Young RH, Norris HJ, Main CS, Lawrence WD, Scully RE. Immunocytochemical localization of placental lactogen and chorionic gonadotrophin in the normal placenta and trophoblastic tumors, with emphasis on intermediate trophoblast and the placental site trophoblastic tumor. Int J Gynecol Pathol 1984; 3: 101-21. Berkowitz RS, Golstein DP. The management of molar pregnancy and gestational trophoblastic tumours. En Knapp RC, Berkowitz RS, editores. Gynecologic Oncology, 2nd ed. New York: Mc Graw-Hill 1992, p. 328-38. De Agustn P, Ruiz A, Lpez F, Contreras F. Patologa de la enfermedad trofoblstica. Simposio Enfermedad Trofoblstica 1972; 79-98. Salem S. Ultrasound diagnosis of trophoblastic disease. En: Sanders RC, James AE(Jr) editores. Ultrasonography in Obstetrics and Gynaecology. New York: Appleton-Century Crofts: 1977; p. 255-66.

2.

3.

4. 5.

6. Silverberg SG, Kurman RJ. Tumors of the uterine corpus and gestational trophoblastic disease. En: Rosai J, Sobin LJ, editores. Atlas of tumor pathology: tumors of the uterine corpus and gestational trophoblastic disease, fasc. 3, ser. 3. Washington DC: Armed Forces Institute of Pathology; 1992, p. 219-85. 7. Szulman AE, Surti U The syndromes of hydatiform mole II. Morphologic evolution of the complete and partial mole. Am J Obstet Gynecol. 1978; 132: 20-7.

16.- Paciente de 25 aos, Gesta 1, Para 1. Con dos citologas lesin de alto grado, prueba de Schiller positiva y biopsia de crvix que demuestra carcinoma In Situ. La conducta es: a) b) c) d) Conizacin. Histerectoma total abdominal. Histerectoma y salpingooforectoma bilateral. Electrocauterizacin del crvix.

La conizacin cervical es el tratamiento de eleccin en pacientes con cncer cervicouterino microinvasor y ms si existedeseo de fertilidad. Asimismo, la histerectoma extrafasciales un mtodo adecuado en lesiones de 0.5 a 3 mm de invasin.Adems se propone que, para pacientes con lesiones de 3.1 a 5 mm de invasin, a partir de la membrana inicial sinfactores de mal pronstico como invasin vascular y linftica, sean tratadas con histerectoma extrafascial, ya que en aquellasa las que se realiz linfadenectoma plvica, con este tipode lesin, no se encontr metstasis a ganglios linfticos.

Referencias: Resultados del tratamiento en cncer cervicouterino microinvasor en el Instituto Nacional de Cancerologa de Mxico (1980-1999) 1.- Mestwerdt G. Fruhdiagnose des Kollumkarzinoms. Zentralb Gynaekol, 1947 ;69 :326. 2. - Morrow CP, Curtin JP. Surgery for cervical neoplasia. In Gynecologic Cancer Surgery. New York, Churchill Livingstone, 1996, p 472.3. 3. - Burghardt E, Holzer E. Diagnosis and treatment of microinvasive carcinoma of the cervix uteri. J Obstet and Gynecol 1977; 49:641-653. 4.- Sedlis A, Sall S, Tsukada Y, et al. Microinvasive carcinoma of the uterine cervix: a clinical-pathologic study. Am J. Obstet Gynecol. 1979;133:64.

17.- Paciente que cursa con 38.5 semanas de gestacin que ingresa al servicio con trabajo de parto. En el transcurso de trabajo de parto durante la dilatacin presenta dolor intenso y brusco. A la exploracin usted observa metrorragia escasa y aumento del tono uterino a la palpacin abdominal que resulta muy doloroso. Cul sera su diagnstico?: a) Placenta previa. b) Desprendimiento de placenta. c) Crioamnionitis hemorrgica. d) Rotura de vasos previos.

Fisiopatologa de la hemorragia: La hemorragia es el signo fundamental que domina el cuadro clnico de la placenta previa. Para explicar su mecanismo existen distintas teoras: Mecanismo de Jacquemier: Se produce crecimiento armnico de la placenta y del tero hasta la semana 26, 28. Despus el segmento inferior crece ms deprisa y favorece el despegamiento lo que origina la hemorragia en el embarazo. Mecanismo de Schroeder: Las contracciones uterinas en el parto traccionan del segmento inferior hacia arriba y empujan al feto hacia abajo despegando la placenta. Mecanismo de Pinard: Explica las hemorragias gestacionales y del parto. El estiramiento de las membranas de la zona de menor radio (orificio interno cervical) como consecuencia de las contracciones tira de la placenta y la desprenden. Mecanismo de Bartholomew: Explica la hemorragia en los casos de placenta previa central. La zona placentaria que reviste el orificio interno es un rea isqumica ya que no recibe vasos deciduales. A este nivel disminuye la presin sangunea, por lo que la sangre tiende a dirigirse hacia esta zona y escapa por la cara materna. En el alumbramiento tambin puede haber una hemorragia importante producida por un doble mecanismo: -desprendimiento parcial antes de la expulsin en los casos de placenta oclusiva. -atona uterina en la zona de insercin despus de expulsada la placenta y vascularizacin anmala. Manifestaciones clnicas de la placenta previa en el embarazo - Sntomas: Principalmente la hemorragia. Toda hemorragia vaginal acontecida en el tercer trimestre debe hacer pensar en una placenta previa. Las hemorragias suelen ser espontneas, no acompaadas de dolor, de sangre roja y se presentan de forma intermitente, con intervalos variables entre las mismas. Progresivamente se van haciendo ms frecuentes y ms graves. La primera hemorragia suele aparecer en forma inesperada generalmente nocturna, cesando en menos de media hora. Las hemorragias ulteriores son ms graves y ms precoces. - Signos: La consecuencia fundamental es la anemia materna que depende de la cuanta de la hemorragia (la sangre es de origen materna ya que procede de espacios intervellosos).

- Exploracin: Exploracin general para valorar la existencia de signos de anemia. Exploracin obsttrica: valorar el tamao del tero (adecuado para la edad gestacional), es blando e indoloro. A menudo la esttica fetal est alterada (transverso, oblicuo, nalgas). No debe efectuarse tacto vaginal cuando haya existido hemorragia en embarazo avanzado por el riesgo de infeccin y de despegamiento y aumentar as la hemorragia. Auscultacin fetal normal. Exploracin ecogrfica: es una tcnica fundamental en el diagnstico de la placenta previa. Permite determinar la localizacin placentaria y la variedad de la placenta previa. En general, la placenta puede identificarse a partir de la 9 semana. No obstante, a lo largo de la gestacin por crecimiento uterino se produce un cambio en sus relaciones con el tero ("emigracin placentaria", imagen de desplazamiento). As el diagnstico de certeza de lmites placentarios solo puede establecerse hacia la semana 34. Siempre ser necesario hacer una adecuada identificacin del orificio cervical interno (ms fcil con sonda transvaginal). Aadiendo al estudio Doppler color se observa la vascularizacin y las zonas que sangran.

18.- Paciente de 59 aos de edad con la siguiente sintomatologa: plenitud, estreimiento, imagen qustica en el ultrasonido plvico en ovario derecho de 15 por 15 cms. El diagnstico ms probable es:

a) b) c) d)

Cistadenoma seroso. Disgerminoma. Endometrioma. Teratoma qustico.

Los Tumores de Ovario son una patologa frecuente dentro del contexto de la patologa femenina. Por esta causa consultan un grupo elevado de mujeres, tanto las consultas de ginecologa como las de Ciruga propiamente dicha. Las edades oscilan desde las tempranas hasta las ya avanzadas, siendo el riesgo de degeneracin maligna muy variable y relacionado con le edad. La experiencia de la clnica revela la alta incidencia de tumores de ovario en la etapa del climaterio, comprendida entre los 35 y 65 aos de edad 1.

El cistoadenoma seroso de ovario (CSO) es un tipo de tumor derivado del epitelio superficial (celmico), formado por reas qusticas. El cistoadenoma seroso de ovario es el tumor ms frecuente de aquellos que provienen del epitelio celmico superficial. Hay tumores pequeos macroscpicamente y tumores masivos que ocupan toda la pelvis e incluso la cavidad abdominal. Estas frecuentes neoplasias qusticas uniloculares estn tapizadas

por clulas epiteliales altas, cilndricas y ciliadas, llenas de un lquido seroso claro y de superficie lisa con abundantes vasos. Las variedades benigna, limtrofe y maligna representan, en conjunto, 30% aproximadamente de todos los tumores del ovario. El riesgo de presentar tumores epiteliales se incrementa con el paso de la edad, ya que pese a que la declinacin de la funcin ovrica marca el envejecimiento gonadal progresivo, el ovario humano nunca pierde su capacidad para generar tumores. Por lo general, cuando es detectado, su tamao es grande, en donde la imagenologa puede ayudarnos a considerar su diagnstico

Referencia: 1. Captulo 22 Tumores Benignos de Ovario. En: Novak ER, Jones G., Jokes HW. Tratado de Ginecologa. 9 ed. Ciudad de la Habana. Editorial Cientfico Tcnica; 1977.p.432 66. 2. MedlinePlus Enciclopedia Mdica en Espaol: Quistes Ovricos. Disponible en: http://vsearch.nlm.nih.gov/vivisimo/cgibin/querymeta?v%3Aproject=medlineplussp anish&spell=spell&query=Quistes+Ov%C3%A1ricos Acceso: Actualizado 20/6/06. 3. Captulo XL Tumores Ovricos En: Llusi Botella J, Nez Clavero JA. Tratado de Ginecologa. Ciudad de la Habana. Editorial Cientfico Tcnica. 1983; T 3.1; p. 751 803. 4. Seccin 18: Ginecologa y Obstetricia. Captulo 241 Neoplasias Ginecolgicas. Cncer de Ovario. En: Manual Merck. Compendio de Autores. 10 ed. Edicin del Centenario. Barcelona. 1999

19.- Se realiza el diagnstico de mola hidatiforme en paciente de 27 aos de edad El tratamiento inicial indicado en esta patologa es? :

a) b) c) d)

Histerectoma total abdominal. Legrado por aspiracin. Metotrexate y seguimientos radiogrficos. Legrado uterino instrumental.

La mola hidatiforme debe extirparse por completo. En general, el tratamiento de eleccin es el legrado por aspiracin despus de dilatar el cuello uterino. Slo en muy raras ocasiones se realiza una histerectoma. Tras la ciruga, se mide la concentracin de gonadotropina corinica humana para determinar si la extirpacin ha sido completa. Si es as, el valor de esta hormona vuelve a la normalidad, en unas 8 semanas, y se mantiene en esos valores. Si una mujer a la que se le ha extirpado una mola queda embarazada, es difcil interpretar un valor alto de gonadotropina corinica humana, porque podra estar causado tanto por el embarazo como por una parte de la mola que no se ha extirpado. En consecuencia, a las mujeres a las que se les ha extirpado una mola se les recomienda no quedar embarazadas durante un ao. Las molas hidatiformes benignas no necesitan quimioterapia, pero las malignas s. Los frmacos que se usan para este tratamiento son el metotrexato, la dactinomicina o una combinacin de ambos. Referencia: 1. 2. 3. 4. 5. 6. 7. Silverman L,Romero Zambrano F, Saldao S. Enfermedad molar. Diagnstico, tratamiento y seguimiento, 1987. Puertas A, Lpez Fernandez J et al. Enfermedad trofoblstica. Casustica del Hospital Virgen de las Nieves de Granada. Cln Invest Gin Obs 1993; 20: 98-103. Enfermedad trofoblstica gestacional. Propuesta Normativa Perinatolgica y Ginecolgica de Alto Riesgo. Ministerio de Asuntos Sociales. Tucumn. 1996-1997; 19:171-183. Jones. Enfermedad Trofoblstica Gestacional: qu hemos aprendido en la ltima dcada. Am J Gynecol Obstet 1990;162: 1286-1292. Resmen del simposio Enfermedad Troblstica Gestacional del Segundo Congreso Nacional de AGORA, 1990. Gonzalez Merlo et al. Protocolos de diagnstico y tratamiento en Obstetricia y Ginecologa, Barcelona: editorial Salvat, 7:35-45.

20.- Se trata de paciente que cursa con 34 SDG, inicia con datos compatibles de amenaza de parto pre-trmino y ruptura prematura de membranas La principal causa de esta patologa est relacionada estrechamente al siguiente diagnstico: a) Traumatismo. b) Idioptico. c) Infeccin de vas urinarias.

e) Infeccin vaginal por cndida albicans.

La infeccin de vas urinarias es la complicacin infecciosa ms frecuente del embarazo. De hecho, las mujeres son ms susceptibles a la infeccin de vas urinarias debido a los siguientes factores: Una uretra ms corta. Fcil contaminacin de la uretra por bacterias de la vagina y el recto. Posibilidad de la que la mujer no vace por completo la vejiga cada vez que orina. Movimiento de bacterias al interior de la vejiga con cada relacin sexual.

Se suman adems, los cambios que el propio embarazo produce en el aparato urinario como son la relajacin del msculo liso de los urteres que impide que la orina llegue adecuadamente a la vejiga para ser eliminada, y la compresin que ejerce la matriz sobre la vejiga lastimando su cubierta interna y dejndola incapacitada para vaciarse por completo. Bibliografa: 1. Velasco MV. Prevencin y tratamiento del parto pretrmino. Lo nuevo acerca del viejo problema Rev Med IMSS 2001, 39 (%) 417-42. 2. Lastra ELG El parto pretrmino como problema de salud pblica .Perinatol Reprodud. Human. Vol 15 No 2.Abril-junio 2001 113-14.

21.- El sndrome de ovario poliqustico es una enfermedad bien conocida de la mujer en edad frtil y su diagnstico se basa en los siguientes datos:

a) Historia clnica familiar pormenorizada ya que lo ms importante son los antecedentes familiares. b) Realizacin de una Resonancia Nuclear magntica de la hipfisis para descartar un adenoma. c) Estudio ecogrfico ginecolgico con tecnologa tridimensional y biopsia. d) Valoracin clnica, estudio hormonal (andrgenos, FSH y LH) y ecografa ginecolgica.

El sndrome de ovario poliqustico (SOPQ), tambin llamado Sndrome de Stein-Leventhal, es un desorden endocrino causando uno de los desbalances hormonales ms frecuentes en mujeres de edad reproductiva. Para que se considere a una persona con posible SOP, debe cumplir dos de estos tres criterios: oligoovulacin o anovulacin, exceso de actividad poliqusticos (visualizados por ultrasonido ginecolgico). Debe andrognica, ovarios

excluirse: Hiperplasia suprarrenal congnita, tumores que puedan secretar andrgenos y la hiperprolactinemia. DIAGNSTICO En 2003 el grupo de Consenso para el diagnstico del sndrome del ovario poliquistico acord, los siguientes criterios de diagnstico: Oligo y/o anovulacin. Signos clnicos y/o bioqumicos de hiperandrogenismo. Ovarios poliqusticos .

Con exclusin de otras etiologas (hiperplasia adrenal congnita, tumores productores de andrgenos, sndrome de Cushing)

Otros criterios de diagnstico que se han utilizado son: 1. Secrecin inapropiada de gonadotrofina a) Relacin elevada hormona lutenica (LH) a hormona estimulantes del folculo (FSH).

b) Respuesta anormal a la prueba con agonistas de la hormona liberadora de gonadotropinas. 2. Hiperandrogenismo a) Hirsutismo, alopecia andrognica, acn. b) Hiperandrogenemia. I. Testosterona total. II. Testosterona libre (ndice andrgenos libre, etc.) 3. Aspecto de los ovarios a) Ovarios poliqusticos. b) Tamao aumentado (estroma) . 4. Insulino resistencia a) Acantosis nigricans. b) Medicin en ayunas de insulina/glucosa. c) Prueba de tolerancia oral a la glucosa. d) Pruebas dinmicas de sensibilidad a la insulina .

I. Clamp euglicmico II. Test de tolerancia a la glucosa intravenosacon muestreo frecuente de la glucosa o con monitorizacin continua de glucosa 5. Anovulacin crnica a) Historia personal reportada b) Pruebas de funcin ovulatoria I. Grfico de temperatura corporal basal II. Determinacin de LH urinaria III. Determinacin de progesterona srica IV. Biopsia endometrial

REFERENCIAS 1. 2. Nippita, T. A.; Baber, R. J. Premature ovarian failure: a review. Climacteric (Mar2007), Vol. 10 Issue 1, p11-22, 12p Tamai K, Koyama T, Saga T, Kido A, Kataoka M, Umeoka S, Fujii S, Togashi K. MR features of physiologic and benign conditions of the ovary. European Radiology , December 2006, Vol. 16 Issue: Number 12 p2700-2711 The Rotterdam ESHRE/ASRM-Sponsored PCOS consensus workshop group. Revised 2003 consensus on diagnostic criteria and long-term health risks related to polycystic ovary syndrome (PCOS). Hum Reprod . 2004 Jan ;19 (1):41-7. Legro R. Diagnostic Criteria in Polycystic Ovary Syndrome. Semin Reprod Med. 2003 Aug;21(3):267-75 Reaven G. The metabolic syndrome or the insulin resistance syndrome? Different names, different concepts, and different goals. Endocrinol Metab Clin North Am. 2004 Jun ;33 (2):283-303.

3.

4. 5.

22.- Una mujer de 23 aos de edad gesta 1, tuvo un parto vaginal espontneo con un producto con peso de 4,350 g. despus de 5 minutos de traccin suave del cordn umbilical se expuls la placenta, que parece estar intacta. Se inici el masaje del fondo uterino y se pidi a la enfermera que administrara 20 unidades de oxitocina en 100 ml. de solucin Ringer lactato. Despus de una inspeccin cuidadosa del canal del parto se observa una laceracin de segundo grado y una laceracin de 2 cm en la pared vaginal izquierda que se intent reparar. En la exploracin fsica se encuentra un fonso uterino blando y atnico. Los signos vitales son: temperatura 37.1C, TA 164/92, FC 130x, FR 18 X. Cul de los siguientes es el mejor tratamiento? a) Oxitocina 10 unidades directas en goteo intravenoso. b)Metilergonovina 0.2 mg IM. c) Prostaglandina F 0.25 mg IM. d) Legrado.

Referencia: Morgan M, Siddighi S. Ginecologa y obstetricia, National Medical Series. 5 edicin. Mc Graw Hill. Pp. 28.

La atona uterina es la causa ms comn de hemorragia puerperal. El masaje energtico y la oxitocina diluida no han sido tiles para interrumpir la hemorragia y por tanto el siguiente paso es agregar un frmaco uterotnico. La metilergonovina est contraindicada porque la paciente se encuentra hipertensa a pesar de la hemorragia intensa, el siguiente frmaco es la prostaglandina. La administracin de oxitocina no diluida, 10 UI por va IV podra causar hipotensin grave. La exploracin manual podra ser apropiada si se sospecha laceracin como causa de hemorragia. El legrado es apropiado para la hemorragia puerperal tarda, cuando se sospecha retencin de los productos de la concepcin.

23.- Posterior a un trabajo de parto con expulsin normal, y tras una hora aproximada en periodo de alumbramiento en el que se practic masaje uterino y se increment moderadamente la dosis de oxitocina, no se aprecian signos de desprendimiento placentario, se indica una extraccin manual de placenta, que resulta imposible por no existir plano de separacin entre la placenta y la pared uterina. Cul es el diagnstico ms probable?:

a) Engatillamiento placentario. b) Placenta succenturiata con cotiledn aberrante. c) Placenta adherente por acretismo placentario. d) Placenta circunvalata.

Se denomina a la placenta como acreta cuando sta se implanta en zonas donde la decidua es deficiente o anormal y por tanto hay una infiltracin del miometrio por vellosidades coriales; esta infiltracin puede ser focal, parcial o total. A su vez esta condicin se subdivide en acreta, increta y percreta. La placenta increta y percreta infiltran todo el espesor de la pared miometrial, en la percreta adems las vellosidades, perforan la serosa y llegan en algunas ocasiones a infiltrar rganos vecinos, especialmente la vejiga. La PA est limitada a la superficie miometrial. La frecuencia de presentacin del AP vara entre 10 y 48 por 10.000 partos. (Oishi A 1999, Hung TH 1999, Zaki ZM, 1998). En las mujeres con acretismo placentario se han visto factores de riesgo, dentro de los cuales se encuentran: 1. Edad y multiparidad: La presentacin AP, aumenta con la paridad de la paciente y la edad, siendo muy rara en primparas. 2. Placenta previa: esta se ha encontrado en el 30% de los casos de PA. Igualmente se ha visto PA en el 9.3% de las pacientes con placenta previa (Miller DA, 1997).

3. Cesrea anterior, o cirugas uterinas previas: Se ha visto este antecedente en el 25% de los casos. En el 29% de los casos la PA estaba implantada en la cicatriz uterina y solo en el 5% la placenta estaba implantada en otro sitio. (Miller Da, 1997). 4. Dilatacin y legrado, en el 25% de los casos. 5. Infeccin uterina previa, remocin manual de la placenta, leiomiomas y otras anomalas uterinas: La asociacin con estas entidades es inconstante. 6. Niveles anormalmente elevados de feto-protena y de b-HCG, en el segundo trimestrre. (Hung TH, 1999). Una placenta adherente o penetrante no es fcil de diagnosticar antes del alumbramiento. Despus de ste, se manifiesta como retencin placentaria y sangrado uterino. El diagnstico generalmente se realiza, despus de intentar la extraccin manual de la placenta. Las manifestaciones clnicas propias de la placenta adherente, de la placenta acreta y de la placenta increta, consisten en una manifiesta dificultad o imposibilidad para la expulsin o extraccin de la placenta. Como consecuencia de la atona parcial y de la hemostasis insuficiente en las zonas de despegamiento placentario, se producir una hemorragia ms o menos grave que en nada se diferenciar de la hemorragia de la atena uterina. Y no ser solamente al intentar el alumbramiento artificial que se pondran de manifiesto las razones ntimas de la retencin placentaria; alumbramiento que ser engorroso en. la placenta ahderente e imposible en las variedades acreta e increta. En varias ocasiones puede no existir hemorragia y en estos casos la nica manifestacin de este estado morboso ser la prolongacin del perodo del alumbramiento. La placenta adherente, como toda placenta retenida, es pronto presa de un proceso infeccioso sin embargo se han sealado casos de placentas retenidas aspticamente durante muchos mese3. al cabo de los cuales han sido expulsadas sin causar trastorno alguno; pero hay que hacer observar que en estas enfermas se ha tratado de retensin de mebranas por abortes ovulares.

1. 2. 3. 4. 5. 6. 7.

Arredondo-Soberon F, Sabella V, Garza-Leal J, Valente PT. Placenta increta en primer trimestre de embarazo. Ginecol Obstet Mex 1995; 63: 279-81. Cantanzarite V, Stanco L, Schrimmer S et al. Managing placenta previa/accreta. Contemp Obstet Gynecol 1996; 41: 66-95. Ecker JL, Sorem KA, Soodak L, et al. Placenta Increta Complicating a FirsTrimester Abortion A case report. Journal Reproductive Medicine. 1992; 37-10. Finberg G, William J. Placenta accreta: prospective sonographic diagnosis in patients with placenta previa / accreta. Contemp Obstet Gynecol 1996: 41: 66-95. Gist RS, Voung V, Brody S, Rees P, Landry AD. Placenta increta occurring in a bligter ovum. South Med J. 1996; 89(5): 545-7. Harden,MA, Walters MD, Valente PT Postabortal hemorahage due to placenta increta: A case report. Obstet Gynecol. 1990; 75: 523. Hudon L, Belfort MA, Broome DR. Dosis and management of placenta percreta: A review. Obster Gynecol survey 1998; 53: 509-517.

24.- Hace 40 horas una mujer de 19 aos, gesta 1, tuvo un parto con un producto femenino de 3,600 g. la calificacin de Apgar en el RN fue de 9 al primer minuto y 9 a los 5 minutos. La revisin de sus registros de trabajo de parto mostr que tuvo ruptura de membranas 7 horas antes del parto. Sus signos vitales al momento son: temperatura 38.2 C, FC 105 x, TA 110/70, FR 16x; en la exploracin fsica el dolor leve a la palpacin del tero, las mamas se encontraban sin eritema, ni dolor a la palpacin, no haba dolor a la compresin de las pantorrillas. Cul de los siguientes es el mejor tratamiento antes de iniciar los antibiticos?

a) Examen general de orina y cultivo b) Cultivo de secrecin vaginal c) Hemocultivo d) Espirometra por incentivo

El vaciamiento incompleto ocasiona orina residual, distensin vesical excesiva y estasis, adems del cateterismo intermitente con sonda vesical durante el trabajo de parto. Por lo tanto, la vejiga en le puerperio est predispuesta a infecciones. El dolor leve a la palpacin del tero puede ser normal en el puerperio y no se debe suponer de inmediato endometritis puerperal. Cuando se sospecha endometritis, los cultivos de secrecin vaginal tienen poca utilidad porque se encuentran los mimos microorganismos que en mujeres purperas sanas. Los hemocultivos son apropiados para la valoracin diagnstica de la fiebre puerperal, pero no son el paso inicial. La espirometra por incentivo se utiliza en el posoperatorio de inmediato para fomentar la expansin pulmonar y disminuir las atelectasias. El legrado uterino se utiliza para tratar la hemorragia Referencia: Morgan M, Siddighi S. Ginecologa y obstetricia, National Medical Series. 5 edicin. Mc Graw Hill. Pp. 29.

25.- Se presenta paciente de 40 aos de edad a su consultorio refiriendo mastalgia que es ms severa antes de la menstruacin. A la palpacin hay nodularidad excesiva, hiperestesia y reas qusticas que la paciente refiere disminuyen en tamao despus de la menstruacin. El diagnstico ms probable es: a) Mastopata fibroqustica b) Fibroadenomas c) Papiloma intraductal d) Cncer de mama

Es raro encontrar una mujer mayor de 35 aos a quien no le hayan dicho, en un examen fsico mamario, ecogrfico o mamogrfico, que tiene quistes en la mama o que su mama es mastoptica. Es el trastorno benigno de la mama ms frecuente y consiste en un aumento del tejido mamario, especialmente en las zonas superiores y externas de las mamas, hacia las axilas, que las hace ms densas. La mastopata fibroqustica suele presentarse en ambas mamas, aunque puede ser de diferente intensidad en una que en otra.

Puede presentarse a cualquier edad despus del inicio de la menstruacin, pero es ms probable que aparezca entre los 30 aos y la menopausia. Raramente se presenta ms tarde de esa edad.

El origen de este trastorno es funcional y responde a desequilibrios de las hormonas sexuales femeninas y puede condicionar la aparicin de quistes mamarios.

Los sntomas pueden fluctuar de leves a severos en una mastopata fibroquistica mamaria, se acentan tpicamente antes de cada perodo menstrual y desaparecen inmediatamente despus. Los sntomas abarcan: Consistencia de protuberancias (como de "guijarros"), irregular y densa del tejido mamario o generalmente ms notoria en la parte superior externa de la mama Molestia en las mamas o generalmente en ambas mamas o puede ser persistente o puede aparecer y desaparecer Sensacin de llenura en las mamas. Sensibilidad y dolor sordo e intenso. Sensibilidad y edema premenstrual. Secrecin ocasional del pezn .

Bibliografa: Jones III HW, Wentz AC. Tratado de Ginecologa de Novak. Editorial Interamericana-McGraw Hill. Undcima Edicin 1994. DiSaia-Creasman. Oncologa Ginecolgica Clnica. Editorial Mosby. Cuarta Edicin 1994. Van Dinh T. Sumario de Patologa Ginecolgica. Editorial La Prensa Mdica Mexicana. 1992.

Pernoll ML. Diagnstico y Tratamiento Ginecoobsttricos. El Manual Moderno. Mxico. Sexta Edicin 1991. Alvarez-Bravo A. Diagnstico de los trastornos menstruales y hemorrgicos. En: Alfonso Alvarez Bravo y su obra. Editorial Marketing y Publicidad SA. Tomo I. 1993. Vzquez E. Aspectos histoqumicos del endometrio humano despus del tratamiento con progestgenos sintticos. Gac Md Mx 1966; 96: 1277-93. Huerta MR, Malacara JM, Rivera-Cisneros A, Daz Cisneros FJ. Sntomas en adolescentes de dos ciudades de Mxico y su asociacin con el ciclo menstual. Ginec Obstet Mx 1994; 62: 146-50.

26.- Entre las siguientes, la afirmacin correcta respecto a una mujer de 31 aos, gestante de 8 semanas con diabetes mellitus insulino-dependiente de 10 aos de evolucin es:

a) El riesgo de aborto espontneo es elevado, independientemente del grado de control de la glucemia. b) Se debe hacer una determinacin de Hb glucosilada en la primera visita obsttrica (Hb A1). c) Se debe hacer amniocentesis en el segundo trimestre para determinar cariotipo y alfafetoprotena. d) En el segundo trimestre habr que evaluar conjuntamente la alfafetoprotena en suero materno, beta-HCG y el estriol no conjugado.

En los ltimos decenios se ha visto un cambio notable en el pronstico del embarazo cuando ste se asocia a diabetes. Con la introduccin de la insulina en la prctica mdica, hace poco ms de 60 aos, la mortalidad materna se redujo en forma rpida y espectacular, en cambio la morbimortalidad neonatal ha tenido una disminucin lenta y gradual, logrando ser reducida a cifras inferiores al 5%. Gran parte de la mortalidad actual del hijo de madre diabtica, se relaciona con malformaciones congnitas en cuya prevencin se concentra la investigacin actual. El embarazo supone una pesada carga para las diabticas, debido a los efectos diabetognicos que posee en s ste estado. stas pacientes tienen una tendencia a la inestabilidad metablica y precisan monitorizacin frecuente, tratamiento estricto y un ritmo de vida muy controlado. (3) En las pacientes diabticas que ya tienen lesin orgnica, el embarazo puede acelerar el proceso, por lo que precisan un control y un tratamiento intensivo.(3)

Debido a lo anterior es que lo ideal es que reciba asesoramiento antes de que quede embarazada. Importancia de controlar la Glicemia: El control inadecuado de la glicemia suele aumentar la incidencia de abortos en el primer trimestre, anomalas congnitas fetales, macrosoma fetal, polihidroamnios y fetos nacidos muertos. (1)

El perodo de mayor riesgo de malformaciones fetales es el perodo periconcepcional y el de la organognesis, ste, riesgo se puede reducir mediante un adecuado control metablico.(3) A su vez, la administracin de cido Flico antes del embarazo y durante el primer trimestre, en dosis de 400 ug/da, previene la aparicin de defectos del Tubo /Neural Neur Importancia del control fetal durante el embarazo : Debido a la posibilidad de Embriopata Diabtica, es necesario efectuar una determinacin de Hemoglobina Glicosilada en etapas precoces del embarazo, Ecografa Obsttrica a la semana 20 y un Ecocardiograma Fetal a la semana 24 para descartar malformaciones fetales ms habituales.(3)

La HbA1c, tambin conocida como hemoglobina glicosilada o glicada, glucohemoglobina o HbA1, es un trmino utilizado para describir una serie de componentes estables minoritarios de la hemoglobina que se forman lentamente y sin intervencin enzimtica, a partir de la hemoglobina y la glucosa. La velocidad de formacin de la HbA1c es directamente proporcional a la concentracin ambiente de glucosa. Como los eritrocitos son fcilmente permeables a la glucosa, el nivel de la HbA1c en una muestra de sangre facilita la historia glucmica de los 120 das anteriores, duracin media de la vida de estas clulas. En particular, la HbA1c refleja de una forma bastante exacta la glucemia en los 2-3 meses anteriores al anlisis. Bibliografa: 1. Cunningham F, MacDonald P, Gant tN. N. Diabetes. Williams Obstetricia, Editorial Mdica Panamericana,20 0a ed ed.1998;52:1119 .1998;1119-1136. 2. Oyarzn E.Patologa Mdica del Embarazo. Alto Riesgo Obsttrico.Impresos Universitarios S.A, 2 ed ed.1997;177 .190. 3. Arias F.Gua Prctica para el Embarazo y el Parto de Alto Riesgo. Mosby Doyma Libros.Segunda Edicin.1995;15:284 -303. 4. Catalano P, Feudtner C, Rosenn B.Diabetes y Embarazo. Clinicas Obstetricas y Ginecologicas , McGraw Hill Interamericana,Primera Edicin.Volumen 1 /2000;1 1-14.-Guas Perinatales ao 2002.Diabetes y Embarazo.Ministerio de Salud Chile; Pginas:22 39. 6.Garner P. Type I diabetes mellitus and pregnancy .The Lancet 1995;346:157 157-161. 7. Barrett J., Salyer S., Boehm F. The nonsstress test: An evaluation of 1000 patients Am J Obstet Gynecol 1981;141:153 -157. 8. Dicker D., Feldberg D., Yeshaya A., Peleg D., Karp M., Goldman J. Fetal surveillance in insulin dependent diabetic pregnancy :Predictive value of the biophysical profile .Am Obstet Gynecol 1988;159:800 8804.

27.- Al encontrarse reparando una de la episiotoma media realizada posterior a la atencin de parto eutcico, en una mujer de 24 aos de edad aprecia un marcado incremento en el sangrado transvaginal. La causa ms probable de este fenmeno es: a) Retencin de restos placentarios. b) Laceracin vaginal. c) Laceracin cervical. d) Atona uterina.

La mortalidad materna es un indicador de disparidad social y econmica. Cada ao en todo el mundo mueren cerca de 600,000 mujeres, entre 15 y 49 aos. Como resultado de complicaciones relacionadas con el embarazo, el parto y el puerperio. Entre las causas principales destacan: hemorragia postparto (25 %), se psis (15%), eclampsia (12 %) y labor prolongada o detenida (8 %). DEFINICIN Se define la hemorragia posparto (HPP) como la prdida sangunea de 500 mL. o ms en las primeras 24 horas despus del parto o el descenso del hematocrito en un 10 % o ms. CLASIFICACIN HPP Inmediata.- Prdida sangunea de 500 mL. o ms originada en el canal del parto dentro de las 24 horas posteriores al parto. HPP Tarda.- Sangrado anormal o excesivo originado en el canal del parto que se presenta entre las 24 horas posteriores al parto y el final del puerperio (42 das). FACTORES DE RIESGO Se han descrito los siguientes factores de riesgo para la HPP: Embarazo mltiple Polihidramnios Macrosoma Trabajo de parto disfuncional Gran multiparidad Corioamnionitis Uso inadecuado de oxitcicos Endometritis en el puerperio Prpura trombocitopnica Anestesia general Administracin previa y reciente de inhibidores uterinos Placenta previa Enfermedad de von Willebrand Desprendimiento prematuro de placenta Acretismo placentario CAUSAS DE HEMORRAGIA POSPARTO A continuacin se lisian las causas ms frecuentes de HPP. Segn su origen, se dividen en uterinas y no uterinas:

Uterinas - No Uterinas Hipotona o atona uterina Retencin de placenta o restosplacentarios o membranas Placentacin anormal (acretismo) Inversin uterina Traumatismo uterino (rotura uterina, desgarro cervical) Laceraciones del canal del parto, incluyendo la episiotoma Coagulopatas Hematomas CUADRO CLNICO La HPP se caracteriza por los siguientes signos y sntomas: Sangrado transvaginal de moderado a grave. tero flcido (no contrado) o desgarros. Alteraciones hemodinmicas que se manifiestan como: mareos, sudoracin, nuseas, taquicardia y/o hipotensin arterial. Oliguria. tero nacido (no contrado). Al considerar la HPP, deben diferenciarse dos tipos de situaciones: la prevencin orientada a minimizar la probabilidad de que una mujer presente hemorragia tras el parto y el manejo o tratamiento de la hemorragia, cuando sta ya se haya producido

Referencias Bibliogrficas
1. AbdaRabbo SA: Stepwise uterine devascularization: A novel technique for management of uncontrollable postpartum hemorrhage with preservation of the uterus. Am J Obstet Gynecol 1994;171:694-700. 2. Bakri YN, Linjawi T: Angiographic embolization for control of pelvic genital tract hemorrhage. Report of 14 cases. Acta Obstet Gynecol Scand 1992;71:17-21. Bick RL: Disseminated intravascular coagulation. Objetive criteria for diagnosis and manegement.- Med Clin N Am Vol 1994;78(3):511-43. B-Lynch C, Coker A, Lawal A II, Abu J, Cowen MC: The B-Lynch surgical technique for the control of massive postpartum haemorrhage: an alternative to? Five cases reported. Br J Obstet Gynaecol 1997;104:372-5. Brahaems D: Unwanted hysterectomies. Lancet 1993;342-61. Braithwaite JL: Variations in origin of the parietal branches of the internal iliac artery. J Anatomy 1952;1:423-30.

3.

4.

5. 6.

28.- Cuando se administra sulfato de magnesio para el tratamiento de la preecampsiaeclampsia y aparecen signos de sobre dosificacin qu antdoto se debe emplear?:

a) Carbonato sdico. b) Nitroprusiato. c) Simpaticomimticos. d) Gluconato clcico.

NIVEL DE PRIMER CONTACTO (ATENCION PRIMARIA) Se debe instruir a todas las embarazadas que deben acudir inmediatamente a un centro de salud en cualquiera de los siguientes casos: Edema que se desarrolla rpidamente (en pocos das) Cefalea severa y persistente. Dolor en la regin abdominal superior. Visin borrosa.

Se debe realizar la medicin de la presin arterial y un anlisis de orina para la deteccin de proteinuria a las mujeres que acudan a centros de salud presentando estos sntomas. Convulsiones Si se asiste a una mujer con eclampsia en un centro de atencin primaria, 1. deben mantenerse las vas respiratorias permeables; 2. se debe colocar a la mujer de costado (posicin decbito lateral izquierda) para evitar la aspiracin del vmito u otras secreciones; 3. si es posible, se debe establecer una va intravenosa; 4. se debe administrar sulfato de magnesio. Monitoreo de la administracin de sulfato de magnesio: Durante el tratamiento con sulfato de magnesio, se recomienda realizar un control cada 4 horas, como mnimo, para detectar la presencia de: Reflejo rotuliano, frecuencia respiratoria superior a 16 por minuto, volumen de orina >100 ml en las 4 horas previas. - Sobredosis de sulfato de magnesio: Todo centro de salud que utilice sulfato de magnesio debe disponer de ampollas de gluconato de calcio (1 g) como antdoto para la sobredosis de dicho frmaco.

Se sugiere medir la presin arterial y administrar antihipertensivos segn corresponda. Convulsiones recurrentes: en caso de convulsiones recurrentes, se administran otros 2 a 4 g de sulfato de magnesio por va IV en el lapso de 5 minutos, tanto para el rgimen IM como el IV; la dosis se determina en funcin del peso de la paciente.

El sulfato de magnesio es un frmaco usado en el control de las convulsiones eclmpticas, para suprimir o controlar las contracciones uterinas sean estas espontneas o inducidas, y como broncodilatador luego del uso de beta agonistas y agentes anticolinergicos. Tambin tiene indicacin como terapia de reemplazo en la deficiencia de magnesio, como laxante para reducir la absorcin de txicos del tracto gastrointestinal. El sulfato de magnesio esta ganando popularidad como tratamiento de inicio en el manejo de algunas arritmias, particularmente en Torsades de Pointes, y en arritmias secundarias a sobredosis de antidepresivos tricclicos o toxicidad digitlica. Esta tambin considerado clase Ila (probable beneficio) para la fibrilacin ventricular refractaria y la taquicardia ventricular, luego de la administracin de dosis de lidocaina y bretilio.

FARMACODINAMIA El sulfato de magnesio tiene la capacidad de alterar la excitabilidad de la fibra miometrial, afecta el acoplamiento excitacin contraccin y el proceso mismo de contraccin, inhibe la entrada de calcio al sarcoplasma y reduce la frecuencia de los potenciales de accin. Inhibe tambin la liberacin de acetilcolina. Por ser estas acciones comunes en las fibras musculares se pueden ver afectadas tambin la musculatura voluntaria e incluso las fibras miocrdicas.(1)

Bibliografa. 1. Graves C. Frmacos que contraen o relajan el tero. En: Hardman J, Limbird L, Molinoff P, Ruddon R, Goodman A, eds. Goodman & Gilman. Las Bases Farmacolgicas de la Teraputica. 9 ed. Mxico DF: McGraw-Hill Interamericana; 1996. pp. 1012-3.

29.- Al encontrase usted en la sala de expulsin atendiendo un parto, observa que se enfrenta ante una atona uterina Cul es el manejo inmediato a realizar? a) Masaje y compresin del fondo uterino. b) 20 unidades IV de oxcitocina. c) 0.2mg Im de metilergonovina. d) Empaquetar con gasas.

Atona Uterina a) Compresin bimanual externa. b) Masaje uterino combinado (endouterino y abdominal). c) Medicamentos: uso de oxitcicos. d) Evacuacin uterina de cogulos. e) Transfusin de sangre o hemoderivados. f) Evaluar factores de coagulacin. g) De persistir el cuadro clnico, proceder a: 1. Ligadura de arteria uterovrica. 2. Sutura de las paredes uterinas (puntos de colchonero). 3. Sutura de Lynch. 4. Ligadura bilateral de arterias hipogstricas. 5. Histerectoma abdominal subtotal. 6. Histerectoma abdominal total. h) Hemoglobina - hematocrito control seriado.

Referencia:
(1) Gua Clnica Basada en las Evidencias: Manejo de la Hemorrasia Postparto. Centro Latinoamericano de Perinatologia y Desarrollo Humano (CLAP). Novedades del CLAP no 16, abril 2002. (2) OMS/FNUAP/UNICEF/BM. Integrated Management Of Pregnancy And Chidbirth IMPAC, 2000. (Traduccin al Espaol: Manejo de las complicaciones del embarazo y el parto: Gua para obstetrices y mdicos 2002 (3) OutLook. Prevencin de la Hemorragia Postparto: Manejo del Tercer Perodo del Parto.

30.- Se trata de femenino de 33 aos con antecedentes patolgicos de hipertensin arterial crnica bien controlada tratada con IECAS , actualmente cursa con 7 semanas de gestacin , signos vitales dentro del parmetro normal y exmenes de laboratorio sin alteraciones, se refiere asintomtica , Cul sera la conducta a seguir?.

a) Mantener el tratamiento dado el buen control tensional. b) Mantener el tratamiento y asociar alfametildopa para disminuir los riesgos fetales de los IECAs c) Mantener el tratamiento y asociar hidralacina para disminuir los riesgos maternos de los IECAs. d) Suspender los IECAs dado el riesgo que presentan para el feto.

El uso de IECA y ARAII durante el segundo y tercer trimestre de embarazo est contraindicado, debido a que estos medicamentos inducen toxicidad fetal (descenso de la funcin renal, oligohidramnios, retraso en la osificacin del crneo) y toxicidad neonatal (insufi ciencia renal, hipotensin, hiperpotasemia). En cuanto a su uso durante el primer trimestre de embarazo, un estudio publicado en el ao 20061 mostraba un incremento de la incidencia de malformaciones congnitas, en particular malformaciones cardiacas, en nios nacidos de madres expuestas a IECA durante el primer trimestre de embarazo en comparacin con las mujeres que no recibieron tratamiento antihipertensivo o que recibieron tratamiento con otros medicamentos antihipertensivos. Estudios posteriores realizados no han confirmado a da de hoy los resultados de este estudio. En lo referente a los ARAII, no se dispone de estudios epidemiolgicos analticos apropiados, por lo que no se puede descartar que exista el mismo riesgo que para los IECA. A pesar de estas incertidumbres, el Comit de Medicamentos de Uso Humano (CHMP) de la Agencia Europea de Medicamentos (EMEA) ha recomendado prudencialmente evitar el uso de IECA

y ARAII durante el primer trimestre del embarazo.

Referencia:
Cooper WO et al. Major congenital malformations after fi rst-trimester exposure to ACE inhibitors. N Engl J Med 2006; 354 (23): 243- 51. (ref.: 2008/10, junio.

31.- Se trata de femenino de 23 aos, G1 en trabajo de parto prematuro con embarazo de 30 semanas de gestacin. A pesar del uso de agentes tocolticos, estos no han dado resultado. Se puede inducir la maduracin pulmonar del producto por medio de: a) Betametasona. b) Sulfato de magnesio. c) Hidroxiprogesterona. d) Clorprocana.

La utilizacin de betametasona como inductor de madurez pulmonar fetal (IMPF) disminuye la morbilidad neonatal relacionada con prematurez pero su efecto diabetgeno materno ha sido poco estudiado. La revisin Cochrane de un ciclo nico de corticosteroides se actualiz en 2006. En esta actualizacin se incluyeron 21 estudios con un total de 3885 mujeres y 4269 lactantes. En la revisin se descubri que la administracin de determinados corticosteroides a mujeres con riesgo de tener un parto prematuro reduce considerable los riesgos de complicaciones relacionadas con la prematurez como muerte fetal y neonatal combinada, sndrome de dificultad respiratoria, hemorragia cerebroventricular, enterocolitis necrotizante, infecciones sistmicas y retraso en el desarrollo durante la niez. Los beneficios estaban presentes cuando el tratamiento se iniciaba entre las 26 y las 35 semanas de gestacin y en los nios que nacan entre 1 y 7 das despus de haber comenzado el tratamiento; tambin se observaron beneficios en los subgrupos de mujeres con rotura prematura de membranas y trastornos hipertensivos. La muerte fetal y neonatal combinada se redujo incluso en neonatos que nacieron a menos de las 24 horas de haber administrado la primera dosis. No se demostraron beneficios cuando el tratamiento comenz antes de las 26 semanas de gestacin, tampoco se observaron beneficios en los recin nacidos antes de las 26 semanas de gestacin ni en los que nacieron despus de 7 das o ms de la administracin del tratamiento. En el caso de los neonatos que nacieron despus de las 36 semanas hubo una tendencia a aumentar la muerte fetal y neonatal combinada. Se observ una reduccin en el peso al nacer en los neonatos que nacieron entre los das 1 y 7, al igual que en los que nacieron ms de 7 das despus del primer tratamiento. Un estudio que reclut mujeres con preeclampsia severa sugiri que las mujeres tratadas tenan un mayor riesgo de sufrir diabetes gestacional.

La evidencia epidemiolgica y en animales sugiere que pueden haber efectos adversos a largo plazo por la exposicin prenatal a los corticosteroides, entre ellos la alteracin de la tolerancia a la glucosa y la hipertensin. Los estudios en animales tambin han sugerido que afecta el crecimiento del cerebro. 1. 2. National Institute of Health (NIH). Consensus Conference; Effect of corticosteroide for fetal maturation on perinatal outcomes. JAMA 1994;(12):1-19. White A, Marcucci G, Andrews E, Edwards K. Antenatal steroids and neonatal outcomes in controlled clinical trials of surfactant replacement. Am J Obstet Gynecol 1995; (173):286-90. Klauss MH, Fanaroff AA, Martin RJ. Problemas respiratorios. En: Asistencia del recin nacido de alto riesgo. 2 ed. La Habana: Editorial Cientfico-Tcnica, 1981:194. Avery M, Frank N, Gribetz I. The inflationary force produced by pulmonary vascular distention in excised lungs. The possible relation of this force to that needed to inflatc the lungs at birth. J Clin Invest 1959;38:456. Chu J, Clements J, Cotton E. Neonatal pulmonary ischemia. Pediatrics 1965;40:733. Liggins GC, Howle RN. A controlled trial of antepartum glucocorticoid treatment for prevention of respiratory distress syndrome in premature infants. Pediatrics 1972;50: 515-25. Wright LL, Verter J, Younes N. Antenatal corticosteroids administration and neonatal outcome in infants 501 to 1500 g. Am J Obstet Gynecol 1995; (173):263.

3.

4.

5. 6.

7.

32.- Gestante de 9 semanas la cual tiene contacto con un menor que, 6 das ms tarde, desarrolla un cuadro de exantema y sndrome general infeccioso sugerente de infeccin por virus de rubola. En el primer control serolgico gestacional se detect la negatividad de la IgG especfica. Cul de las siguientes afirmaciones es la correcta?:

a) La aparicin de IgM materna positiva constituye indicacin para la determinacin de la IgM fetal. b) No existe posibilidad de contagio dado que el nio ya no se hallaba en fase de eliminacin viral. c) En este perodo de la gestacin, el riesgo de la infeccin y de afectacin embrionaria es mnimo. d) La existencia de IgM fetal negativa excluye en este caso la posibilidad de transmisin transplacentaria.

INFECCIONES EN EMBARAZADA VACUNACIONES: CONTRAINDICADAS: - Parotiditis. - Rubola. - Sarampin. - Fiebre amarilla.

- NO se RECOMIENDA: gripe, poliomielitis, hepatitis B. S SE PUEDEN DAR (cuando estn indicadas): - Fiebre tifoidea. - Rabia. - Ttanos. - Tos ferina. (ver pregunta mir, que pone que no) TOXOPLASMOSIS: 50% de transmisin. Si grave (en 1er T, pero es menos frecuente): aborto, parto pretrmino, muerte fetal intratero. Dx: Lo de siempre. Calcificaciones cerebrales. TTO: - En casos de seroconversin ESPIRAMICINA (depresin medular, ac. folnico) ht final del embarazo. - Si infeccin fetal: PIRIMETAMINA Y SULFADIAZINA en ciclos de 3 semanas alternando con la espiramicina ht final del embarazo. RUBEOLA: Contagio >80% si la madre se contagia en 8 primeras semanas. SORDERA CONGNITA. Acs <1/16 susceptibilidad para infeccin. La aparicin de IgM materna positiva es indicacin de determinacin de IgM fetal. La madre no debe quedarse embarazada en los 3 meses siguientes a la vacunacin. Profilaxis de EXPOSICIN: gammaglobulina, solo eficaz en primeros 7-8d. CITOMEGALOVIRUS: Insospechada. Inclusiones citomeglicas en OJO DE BHO en clas de tejidos afectados. SFILIS: Prueba sexolgica a todas las gestantes. Si se infecta, en los 3 meses neonatales: lesiones CUTNEOMUCOSAS, OSTEOCONDRITIS Y HEPATOESPLENOMEGALIA. DX: screening VDRL o RPR (no treponmicas, pero el embarazo es la primera causa de falso positivo) certeza FTA-Abs o MHA-TP (treponmicas) VARICELA: Si en 1er T 2%; gravemente teratognica. En perodos ms avanzados o periparto enfermedad sistmica generalizada o SNC.

HEPATITIS B: Se infecta el hijo si: - Madre PORTADORA CRNICA - INFECCIN ACTIVA durante la gestacin - Madre con HEPATITIS CRNICA ACTIVA HBsAg positivo (slo) riesgo bajo de transmisin placentaria HBsAg + HBeAg 90% de transmisin *El riesgo de cronificacin ser muy alto si se adquiere en el perodo perinatal.

* Especial vigilancia del crecimiento fetal. *Se realizar profilaxis ACTIVA y PASIVA *La gestacin NO aumenta el riesgo de curso clnico grave.

RUBOLA Y EMBARAZO Existen tres situaciones claramente diferenciadas que exigen planteamientos diagnsticos distintos: Determinacin de la inmunidad frente a rubola en la gestante, sin sospecha clnica ni epidemiolgica de padecer la enfermedad: El objetivo de este estudio es conocer si la gestante est protegida, de una posible infeccin por el virus de la rubola, durante el embarazo. Se recomienda la determinacin cualitativa de anticuerpos totales o de IgG especfica, en la primera consulta de control del embarazo. Se desaconseja expresamente la evaluacin cuantitativa de los resultados, ya que no proporciona ninguna informacin til. La presencia de anticuerpos refleja contacto previo con el virus, y por tanto inmunidad, haciendo innecesaria la realizacin de nuevos controles en embarazos sucesivos. A pesar de que se describe que la rubola puede cursar de manera asintomtica, F. de Ory et al estudian 185 sueros de 101 mujeres embarazadas con presencia de IgM y slo confirman la existencia de primoinfeccin en tres de las mismas, asocindose siempre a datos clnicos o epidemiolgicos compatibles, por lo que es desaconsejable la realizacin sistemtica de IgM a las embarazadas. Si la mujer embarazada es seronegativa, deber adoptar las precauciones necesarias para evitar la exposicin al virus y debe ser vacunada frente a la rubola en el post-parto inmediato. Sospecha clnica de infeccin aguda durante el embarazo: Este caso puede plantearse ante la existencia de una clnica compatible en la embarazada, o por exposicin a un sujeto con infeccin aguda por rubola. La presencia de IgG en ausencia de IgM indica que la mujer est protegida, por vacunacin o por infeccin antigua y por tanto no deben realizarse ms determinaciones. La demostracin de seroconversin, con ausencia de anticuerpos en el primer suero y presencia de stos en el segundo, obtenido 15-21 das despus, es la forma ms segura de diagnosticar una primoinfeccin por este agente. Sin embargo, si el primer suero de la enferma presenta anticuerpos, aunque se produzca un incremento del ttulo de estos en el segundo suero, puede ser debido a una reinfeccin. La presencia de IgG y de IgM especfica en una paciente, nos hace sospechar la presencia de primoinfeccin, sin embargo, debemos tener en cuenta varios aspectos: a.- La IgM puede tener reacciones heterlogas entre rubola y otros virus como EBV, CMV, Parvovirus B 19 y virus del sarampin (por reacciones cruzadas o por estimulacin policlonal

de linfocitos de memoria); por tanto es necesario confirmar su presencia, siendo la tcnica de ELISA de captura la que presenta mejor especificidad y sensibilidad. b.- La IgM puede aparecer durante las reinfecciones, pero a ttulos bajos y durante poco tiempo. c.- En un pequeo porcentaje de personas, la IgM puede mantenerse positiva en suero hasta 6 meses. Thomas et al, detectan la presencia de IgM en el 9% de los casos a los 3 4 meses de la infeccin aguda. El estudio de la avidez de la IgG diferencia si la IgG es de aparicin reciente (baja avidez se asocia a infeccin primaria aguda) o si hay ausencia de infeccin primaria (IgG de alta avidez), puede ser una tcnica que ayude a valorar la presencia de IgM y puede colaborar en la diferenciacin entre primoinfeccin y reinfeccin. F. de Ory et al. estudian mltiples patgenos y comunican que esta tcnica presenta una sensibilidad entre el 81 y el 100% y una especificidad del 100%. Tambin se est valorando la utilidad en el diagnstico de la IgA, aunque los datos no son an concluyentes. Todos estos datos serolgicos deben ser interpretados junto con los datos clnicos de la embarazada, en el caso de que los haya y junto con los datos que podamos obtener de la posible fuente de infeccin

BOSMA TJ, CORBETT KM, OSHEA S, BANATVALA JE, BEST JM. PCR detection of rubella virus in clinical samples. J Clin Microbiol 1995; 33:1075-1079. BOSMA TJ, CORBETT KM, OSHEA S et al. Use of PCR for prenatal and postnatal diagnosis of congenital rubella. J Clin Microbiol 1995; 33:2881-2887. DE ORY F, CASAS I, DOMINGO CJ, ECHEVARRA JM. Application of fluoroimmunoassay to the identification of low avidity specific IgG against pathogenic human viruses and Toxoplasma gondii. Clin Diagn Virol 1995; 3:323-332. DE ORY F, DOMINGO CJ. Los anlisis de avidez de la IgG especfica en el diagnstico de la infeccin por el virus de la rubola. Med Clin (Barc) 1996; 107:118. DE ORY F, ECHEVERRA JM, DOMINGO CJ. Cribado rutinario de IgM especfica antirrubola en mujeres embarazadas: una prctica desaconsejable. Prog Obstr Ginec. Dic 1998; 41:574-578. ENGLUND J, GLEZEN WP, PIEDRA PA. Maternal immunization against viral disease. Vaccine 1998; 16:1456-1463. FREY TK, ABERNATHY ES, BOSMA TJ et al. Molecular analysis of rubella virus epidemiology across three continents: North America, Europe and Asia, 1961-1997. J Infect Dis 1998; 178:642-650.

33.- Mujer de 23 aos G1 con 5 semanas de gestacin tom cumarina hasta el da en que se enter que estaba embarazada. Est preocupada de que la cumarina ocasione defectos congnitos en el producto, por lo que usted le informa que el feto es ms susceptible a teratognesis durante el siguiente periodo: a) Durante el segundo y tercer trimestre b) De la ovulacin a la implantacin c) Entre la implantacin y el da en que se espera la menstruacin d) Durante las primeras 12 semanas de la gestacin La administracin de frmacos a una mujer embarazada requiere una cuidadosa evaluacin, ya que muchos de estos frmacos pueden alcanzar al embrin o al feto, colocandolo en una situacin de riesgo de sufrir defectos congnitos. Por esta razn, debe sopesarse cuidadosamente este riesgo potencial con el beneficio aportado a la madre por esa medicacin. Una mujer gestante puede llegar a recibir ms de 11 medicaciones diferentes durante su embarazo. Se calcula que alrededor del 35% de las mujeres gestantes (para otros hasta el 80%) han tomado cualquier tipo de frmaco en este periodo, bien por prescripcin facultativa o automedicacin. Se estima que un 2-4% de los nacidos vivos tienen anomalas congnitas en forma de anomalas estructurales mayores, aumentando el porcentaje a 810% si se alarga la vigilancia hasta los 5 aos, debido a que los teratgenos funcionales suelen manifestarse ms tardiamente. A pesar de su importancia, tanto desde el punto de vista mdico como social, se desconoce la causa del 65-70% de los casos de malformaciones congnitas. Las enfermedades hereditarias constituyen el 15-20%, los transtornos cromosmicos 5% y factores ambientales como la exposicin a frmacos 2-4%; infecciones congnitas y enfermedades sistmicas causan el resto. Etapa del desarrollo en el momento de la exposicin:

Los efectos teratognicos de un frmaco en el embrin en desarrollo o en el feto se cree que dependen sobre todo de la edad gestacional, o sea de la etapa del desarrollo en el momento de la exposicin. Podemos considerar varias etapas en el desarrollo: - Periodo de prediferenciacin (0-2 semanas): incluyendo el periodo de implantacin; existe una baja susceptibilidad del embrin a las acciones teratognicas, ya que, debido al carcter totipotencial de las clulas embrionarias, si una clula se destruye otra puede tomar su funcin. La exposicin, en este periodo, sigue la ley del "todo o nada": o se afecta totalmente, produciendose un aborto o resorcin, o no hay lesin. - Periodo de organognesis (3-8 semanas): es el periodo de mxima susceptibilidad a los teratgenos ya que las clulas embrionarias han perdido su carcter totipotencial y se estan formando los diferentes rganos.

- Periodo de histognesis (8-32 semanas): existe una menor susceptibilidad a teratognesis, aunque si existe cierto riesgo funcional. - Periodo de maduracin funcional (hasta la semana 38): todava menor riesgo teratognico y funcional.

Tabla I
Frmacos cuya teratognia se ha comprobado en humanos
FRMACO ANTIBIOTICOS Tetraciclinas Tincin dental, hipoplasia enamel. Posible retraso crecimiento seo. Alrededor de 50% de los expuestos a tetraciclina; 12,5% de expuestos a oxitetraciclina. MALFORMACIONES DETECTADAS RIESGO

ANTICOAGULANTES Cumarinas Sndrome fetal de la warfarina: hipoplasia nasal, condrodisplasia punctata, braquidactilia, defectos craneales, orejas anomalas, ojos malformados, malf. en sistema nervioso central, microcefalia, hidrocefalia, deformidades esquelticas, retraso mental, atrofia ptica, espasticidad; malformacin de Dandy Walker. El 16% tiene malformaciones, un 3% hemorragias. Se producen hasta un 8% de abortos.

ANTIEPILEPTICOS Carbamacepina Fenitoina Aumento del riesgo de sufrir defectos del tubo neural. Sndrome fetal de la fenitoina: aplanammiento puente nasal, pliegues epicantales internos, ptosis, estrabismo, hipertelorismo, orejas anormales o de baja implantacin, hipoplasia de falanges distales y uas, anomalas esquelticas, microcefalia y retraso mental, deficiencias del crecimiento, neuroblastoma, defectos cardacos, paladar hendido y labio leporino. Sndrome fetal de la trimetadiona: retraso crecimiento intrauterino, anomalas cardacas, microcefalia, labio y paladar hendidos, anomalas en orejas, facies dismorfica, retraso mental, fstula traqueo-esofgica, muerte postnatal. Espina bfida con mielomeningocele, defectos en sistema nervioso central, microcefalia, defectos cardiacos. Se estima un riesgo del 1%. El 5-10% puede sufrir el sndrome tpico. Un 30% parcialmente. Hasta un 7% de descendientes con coeficiente intelectual bajo.

Trimetadiona (Troxidona)

Basndose en casos notificado: 83% y 32% de muerte infantil o neonatal.

Acido valproico

Un 1% de riesgo de defectos del tubo neural.

ANTINEOPLASICOS Alquilantes Busulfan Clorambucil Ciclofosfamida Clormetina Retraso del crecimiento, paladar hendido, microftalmia, hipoplasia ovrica, opacidad corneal, agenesia renal, malformaciones de los dedos, defectos cardacos, otras anomalas mltiples. Entre el 10-50% de los casos pueden presentar malformaciones, aunque este riesgo, extrado de series de casos puede estar exagerado.

Antimetabolitos Aminopterina Azauridina Citarabina Fluorouracilo Mercaptopurina Metotrexato Hidrocefalia, meningoencefalocele, anencefalia, malf. craneales, hipoplasia cerebral, retraso del crecimiento, malformaciones oculares y del odo, malformaciones nasales, paladar hendido, malf. en miembros y dedos. Sndrome fetal de la aminopterina: disostosis craneal, hidrocefalia, hipertelorismo, anomalas de odo externo, micrognatia, paladar hendido. HORMONAS Dietilestilbestrol Descendencia femenina: Adenocarcinoma de clulas claras vaginal o cervical en mujeres jvenes expuestas "in utero" (antes de la semana 18); oligomenorrea, reduccin de las tasas de embarazo, incremento de las tasas de embarazos pretermino, incremento de la mortalidad perinatal y de aborto espontneo. Descendencia masculina: Quistes epididimarios, criptorquidia, hipogonadismo, disminucin de la espermatognesis, estenosis de meato, hipospadias. NEUROLEPTICOS Litio Mayor riesgo de anomala de Ebstein; no se ha detectado un mayor riesgo de otro tipo de malformaciones. Cambios morfolgicos congnitos en epitelio vaginal en el 39% de las expuestas. En exposicin antes de la 18 semana: riesgo de carcinoma 1.4 por 1000 de expuestas. Entre el 7-75% de los casos expuestos, aunque este riesgo, extrado de series de casos puede estar exagerado.

PENICILAMINA Hiperclastosis cutnea (cutis laxa) RETINOIDES SISTEMICOS Pocos casos. Riesgo desconocido.

Isotretinoina Etetrinato

Aborto espontneo, deformidades craneales, de orejas, cara, corazn, extremidades e hgado; hidrocefalia, microcefalia, defectos cognitivos (incluso sin malformaciones aparentes).

Isotretinoina: 38%. El 80% son malformaciones del sistema nervioso central.

TALIDOMIDA Focomelia, amelia, hipoplasia de miembros, defectos cardacos congnitos, malformaciones renales, criptorquidia, parlisis VI par, sordera, microtia, anotia. Alrededor del 20% cuando la exposicin ocurre entre 4-8 semana.

Bibliografa. McBride WG. Thalidomide and congenital abnormalities. Lancet 1961; 2:1358. Lenz W. Thalidomie and congenital abnormalities. Lancet 1962;1:45. Shepard TH. Catalog of Teratogenic Agents. Johns Hopkins University Press London, 1987. Piper JM. Banur C, Kennedy DI. Prescription drugs use in pregnancy in a Medicaid population. Am J Obstet Gynecol 1987; 157:148-156. Salvador J, Martnez-Fras ML, Rodrguez Pinilla E. Consumo de medicamentos por la mujer embarazada en Espaa. Ministerio de Sanidad y Consumo. Madrid, 1989. Grupo de trabajo DUP. Estudio multicntrico sobre el uso de medicamentos durante el embarazo en Espaa (I). Mtodos y caractersticas demogrficas de la poblacin estudiada. Med Clin 1990;95:764-767. Grupo de trabajo DUP. Estudio multicntrico sobre el uso de medicamentos durante el embarazo en Espaa (II). Los frmacos utilizados durante la gestacin. Med Clin 1991;96:11-15. Grupo de trabajo DUP. Estudio multicntrico sobre el uso de medicamentos durante el embarazo en Espaa (III). Los frmacos utilizados durante el primer trimestre de la gestacin. Med. Clin 1991;96:52-57. Martnez-Fras ML. Medicamentos y teratogenia. Revisin bibliogrfica y situacin en Espaa. Ministerio de Sanidad y Consumo. Madrid, 1989.

34.- Cul de las siguientes afirmaciones, relativas a la ictericia recurrente del embarazo, es correcta?:

a) El 25-30% de los casos evolucionan hacia un hgado graso agudo del emvarazo. b) Probablemente se debe a un exceso de estrgenos y gestgenos. c) Cursa con elevacin significativa de los niveles sricos de fosfatasa alcalina y de colesterol. d) Para evitar que evolucione hacia el hgado graso agudo, deben usarse corticoides a dosis altas.

La Colestasis Gravdica del embarazo es una hepatopata que suele ocurrir en el segundo o tercer trimestre del embarazo. Al formar parte de las enfermedades hepticas y gestacin, hemos realizado una pequea introduccin de las mismas para que resulte ms fcil centrarnos en el tema. ENFERMEDADES HEPTICAS Y GESTACIN: Introduccin Las enfermedades hepticas en el embarazo comnmente se han dividido en: o o o Aquellas que se presentan durante el embarazo y que estn especficamente relacionadas con la gestacin. Enfermedades hepticas crnicas sobre las que sobreviene el embarazo. Las que complican el embarazo como sucede con la hepatitis viral aguda o la insuficiencia heptica inducida por drogas.

Por ello antes de entrar de lleno en la revisin de la colestasis gravdica, vamos a concretar en que grupo de enfermedades hepticas se encuentra. Fisiologa heptica del embarazo El hgado durante la gestacin sufre algunos cambios similares a los que ocurren en pacientes con hepatopata. En el examen fsico hay que destacar la posible presencia de araas vasculares (66% y 14% de gestantes de raza blanca y negra respectivamente), as como de eritema palmar, ambos secundarios al hiperestrogenismo mantenido, sin que ello traduzca una hepatopata avanzada subyacente. En embarazos sin complicaciones, las pruebas de funcionamiento heptico difieren en algunos parmetros de las que se encuentran en la no gestante.

Tabla 1.VARIACIN DE LOS PARMETROS ANALTICOS DURANTE LA GESTACIN Transaminasas (AST o GOT, ALT o GPT) g-glutamiltranspeptidasa (GGT) Fosfatasa Alcalina Bilirrubina cidos biliares sricos 5-nucleotidasa Colesterol Triglicridos Protenas totales =o (2 veces) = =o ( = 2 a 4 veces)

Albmina Transferan Ceruloplasmina Alfa 1 aantitripsina -globulinas Tiempo de protombina Fibringeno Hematocrito Volumen plasmtico = =

( 10 a 60 % )

=o

(50%)

El perfil BQ muestra como principal variante de la normalidad, un ascenso de la fosfatasa alcalina, que se evidencia principalmente en el tercer trimestre del embarazo; esta elevacin no denota un problema colestsico, sino que principalmente, se debe a un aumento de las isoenzimas placentarias y a un mayor metabolismo seo. Los niveles de transaminasas, bilirrubina total, cidos biliares sricos y gammaglutariltranspeptidasa son rigurosamente normales, aunque se sabe que en la embarazada hay un retraso en la eliminacin biliar. Atribuyndose el prurito gestacional a una retencin de sales biliares, comprobable, aunque no exista ictericia. Los parmetros de la coagulacin tambin son normales. Es frecuente observar un discreto grado de anemia por hemodilucin (incremento mayor de la volemia respecto a la masa eritrocitaria). El resto del perfil BQ puede mostrar una hiperlipemia mixta as como una hipoalbuminemia dilucional. La biopsia heptica sin embargo, resulta normal. Los estudios de imagen (ecografa) no revelan alteraciones patolgicas, salvo la posible presencia al final del embarazo de una colelitiasis asintomtica As, en el siguiente cuadro se resumen las enfermedades hepticas que pueden producirse en el transcurso de un embarazo, diferenciando aquellas en las que la gestacin ocurre en el curso de una hepatopata crnica o aguda de aquellas en la que la hepatopata es consecuencia del estado gravdico.

En esta revisin, como ya hemos dicho anteriormente, nos hemos centrado en la Colestasis gravdica del embarazo.

BIBLIOGRAFA: 1.-Tadataka Yamada .Gastroenterologa McGraw-Hill Interamericana 2000 2.-Joaqun Berenguer. Gastroenterologa y Hepatologa. Tercera Edicin. 2002 3.-Lee M Isselbacher KJ. Ictericia En principios de Medicina Interna Harrison 16 Edicin McGraw Hill 2002. 4.- Roche SP, Kobos R. Jaundice in the adult patients. Am Fam Physican 2003; 69: 299304. 5.- Molano RRA, Gallegos OJF. Estudio del paciente ictrico. En Villalobos JJ, Valdovinos MA, Olivera MA. Principios de Gastroenterologa. Mxico: Mndez ed. 2004; pp: 369-86. 22

35.- Mujer de 32 aos que cursa en ste momento con diagnstico de preclampsia leve, el frmaco de eleccin que se administra en esta patologa es: a) b) c) d) Nifedipina. Inhibidores de la enzima convertidora de angiotensina. Clonidinas. Alfametildopa.

Prevenir complicaciones a corto plazo de las mujeres con PA elevada que comprometa el bienestar fetal Cuando la PAS es mayor o igual a 150 mmHg y la PAD mayor o igual a 100 mmHg. El propsito es alcanzar cifras de TA alrededor de 140/90. La medicacin antihipertensiva se reserva para los casos en que la PAD 100 mmHg. Se recomienda continuar el tratamiento antihipertensivo previo al embarazo, exceptuando el uso de IECA. La alfametildopa y la hidralazina va oral son los frmacos de eleccin dado su uso extensivo con seguridad y eficacia y sin efectos colaterales para el feto (excepto hidralazina en lupus).

ALFA METILDOPA 500-2000 MG/DA HIDRALAZINA 50-200 MG/DA LABETALOL 100-400 MG/DIA ATENOLOL 50-200 MG/DA NIFEDIPINA 10-30 MG/DA Referencia: 1. Aagard K, Belfort M. Eclampsia: Morbility, mortality, and management. Clin Obstet Gynecolol. 2005; 48: 12-23. 2. Oyarzn E. Sndrome hipertensivo del embarazo en Oyarzn E. Ed. Embarazo de alto riesgo. Ediciones Universidad Catlica de Chile. Santiago. 1997: 157175. 3. Roberts J, Redman C. Pre-eclamsia: More than pregnancy induced hypertens

36.- Mujer con diagnstico de amenorrea que acude con resultados de laboratorio los que reportan los siguientes niveles hormonales: GnRH elevada, FSH y LH elevadas, hormonas ovricas (estrgenos y progesterona) bajas. El defecto est a nivel de: a) b) c) d) Hipotlamo. Hipfisis. Ovario. Endometrio.

Evaluacin de la paciente con amenorrea secundaria La mayora de las pacientes con AS que no estn embarazadas o no entraron en el climaterio tienen una alteracin en algn nivel de la cascada reguladora del ciclo menstrual femenino. A continuacin, esquematizamos la evaluacin de la AS en pasos. Cada mdico deber adaptar los tiempos de la evaluacin a la situacin particular de cada paciente. No obstante, recomendamos no saltear ninguno de estos pasos ya que un diagnstico preciso permitir realizar un tratamiento racional y dar un pronstico respecto de la futura funcin menstrual y de la fertilidad. Primer paso (descartar el embarazo) Su prueba se basa en el dosaje cuali o cuantitativo de la subunidad beta de la gonadotrofina corinica humana. Si se certifica la ausencia de embarazo podr avanzarse al siguiente paso, pero teniendo en cuenta siempre que si la probabilidad de embarazo es muy alta se deber repetir la prueba nuevamente. Se recomienda no obviar esta prueba aunque la paciente asegure que no ha mantenido relaciones sexuales. Segundo paso (prueba de progesterona) Debe realizarse slo si el test de embarazo es negativo y consiste en dar medroxiprogesterona entre 30 a 50mg por va oral (un comprimido de 10mg durante 5 das) o progesterona oleosa 100 a 200mg por va intramuscular en una sola dosis. El resultado de esta prueba provee informacin acerca de si se produjeron estrgenos. La prueba se considera positiva si se produce un sangrado luego de 2 a 14 das de la suspensin de la progesterona. La respuesta positiva indica que existe integridad anatmica del aparato genital femenino y que el ovario produce estrgenos. En estos casos, la AS se debe a que no ha habido ovulacin. La causa ms frecuente de AS con prueba de progesterona positiva es la disfuncin hipotalmica leve. En este caso, no se dispara el pico de LH necesario para producir la ovulacin, no se ovula (ciclo anovulatorio), no hay cuerpo lteo y no hay produccin de progesterona. El sistema reproductor queda en un estado folicular, con grados variables de desarrollo folicular, acompaados de concentraciones cambiantes de estradiol. Puede producirse hemorragia en ausencia de ovulacin dado que el endometrio proliferativo sufrir ocasionalmente un desprendimiento parcial en respuesta a estos niveles cambiantes de estradiol. Estos sangrados luego de un ciclo anovulatorio se denominan sangrados uterinos disfuncionales (SUD). Como dijimos, ejemplos frecuentes de situaciones de anovulacin son el estrs, la prdida brusca de peso y el ejercicio intenso. Otras causas menos frecuentes son la hiperprolactinemia, el hipotiroidismo y el sindrome del ovario poliqustico. Otras veces no hay una causa clara (idioptica) de este trastorno. La prueba se considera negativa si NO se produce un sangrado luego de 2 a 14 das de la suspensin de la progesterona, lo que ocurre cuando los niveles de estrgenos son bajos (menores a 40 pg/ml). La causa ms frecuente de prueba negativa es la insuficiencia gonadal de la menopausia. Tercer paso Para avanzar al tercer paso es preciso distinguir si la prueba de progesterona fue positiva o negativa. Una prueba de progesterona positiva hace diagnstico de ciclos anovulatorios. Segn los antecedentes, el examen clnico y la probabilidad previa el mdico podr orientarse hacia cul es el diagnstico. Hay situaciones de estrs, crisis vitales, cambios de peso brusco o

Pacientes

con

prueba

de

progesterona

positiva

ejercicio extremo que ocasionan un trastorno del ciclo aislado. En estos casos, la prueba de progesterona es diagnstica y teraputica. Es decir, la paciente no requiere estudios posteriores. Es conveniente reasegurarla y explicarle que su periodo se normalizar una vez solucionado el problema. En una paciente con trastornos anovulatorios frecuentes, que presenta obesidad, acn e hirsutismo, existe alta sospecha de sndrome de ovario poliqustico (SOP). En este caso, debemos solicitar una ecografa pelviana y un dosaje de LH/FSH o de hormonas masculinas: testosterona libre (To), dehidroepiandrosterona sulfato (DHEA-S) y 17OH progesterona. Si la paciente tiene trastornos del ciclo y galactorrea, se debe solicitar un dosaje de prolactina (PRL). El valor normal de PRL para la mujer no embarazada es de 20 a 25ng/ml. Cuando el valor de PRL es menor de 100ng/ml, generalmente es de causa idioptica o farmacolgica; si est entre 100 y 200ng/ml, puede ser por causa farmacolgica o por enfermedad hipotalmica; si es mayor de 200ng/ml, la causa ms probable es el adenoma hipofisario. Cuando el valor de la PRL es mayor de 100ng/ml, se debe solicitar un estudio por imgenes que puede ser una tomografa computada de cerebro con contraste o una resonancia magntica nuclear con gadolinio. Si se observa una imagen menor de 10mm, se trata de un microadenoma de hipfisis y si es mayor de 10mm, se hace diagnstico de macroadenoma. Si la paciente consulta por trastornos del ciclo frecuentes y no presenta galactorrea o signos o sntomas de hipotiroidismo, igualmente deber solicitarse un dosaje de tirotrofina (TSH) y de prolactina (PRL) sricas.

Pacientes

Deber dosarse el nivel srico de FSH cuyo valor refleja los niveles de estrgenos circulantes (por el fenmeno de retroalimentacin negativa). Si la FSH es mayor de 40 UI/ml, los niveles de estrgenos son bajos. En este caso, se asume que existe una insuficiencia ovrica. El valor de la FSH puede hacer el diagnstico diferencial entre la falla ovrica (castracin temprana, menopausia precoz o menopausia normal) y la disfuncin hipotlamo-hipofisaria severa. En la primera, la ausencia de retroalimentacin negativa estimula la liberacin de las gonadotrofinas y, en consecuencia, stas se encuentran elevadas en la sangre. En la segunda, los valores de gonadotrofinas son bajos debido a un defecto en su produccin. El dosaje de FSH es ms sensible que el de LH para este propsito, por lo tanto, si la prueba de progesterona es negativa y la FSH es baja, lo ms probable es que la causa sea una disfuncin hipotlamo-hipofisaria severa. En este caso debera solicitarse un estudio de diagnstico por imagen del cerebro (RMN con gadolinio o TAC con contraste). En el cuadro 4 se muestran los valores normales de las hormonas mencionadas. Cuadro 4: valores hormonales PRL (prolactina): 20 a 25ng/ml en mujeres no embarazadas. normales

con

prueba

de

progesterona

negativa

TSH: de 0.5 a 5mUI/ml.

FSH: 5 a 30mUI/ml.

LH: 5 a 20mUI/ml (en el pico ovulatorio este valor se debe multiplicar por dos o tres). Falla ovrica: FSH mayor de 40mUI/ml.

Disfuncin hipotlamo hipofisaria: FSH menor a 5mUI/ml y LH menor a 5mUI/ml. En general, una paciente con disfuncin hipotalmica con gonadotrofinas bajas est expresando una gravedad mayor que la disfuncin hipotalmica leve con prueba de progesterona/positiva. En los casos en los que la prueba de progesterona es negativa, la FSH es normal, la paciente no est en una edad cercana a la menopausia y existe alta sospecha de que exista una enfermedad uterina, se recomienda realizar una prueba de estrgenos y progesterona. Esta prueba sirve para conocer si el efector (el tero) responde a los estmulos hormonales. Consiste en administrar estrgenos (1.25mg por da durante 20 das) ms progesterona (10mg por da durante los ltimos 5 das en que se administran los estrgenos). Si no hay sangrado, se debe repetir la prueba y, si nuevamente no hay sangrado, la prueba se considera negativa. Esto indica que hay una falla en el efector, o sea, en el tero. En este caso la paciente debe ser derivada al gineclogo. Las causas ms probables de falla uterina son la endometritis por abortos o partos spticos y las sinequias por curetajes vigorosos. Cuando falla el efector, la ecografa transvaginal puede ser normal. Por eso debe realizarse una prueba de estrgenos y progesterona para evaluar directamente la funcionalidad del tero. La prueba se considera positiva si hay sangrado (aunque sean slo gotitas). Esto significa que el tero est indemne para responder a estmulos hormonales y el problema es de origen hipotlamo- hipofisario 1. Jonathan R, Pletcher, Gail B. Slap. Menstrual Disorders Amenorrhea. Pediatric Clinics of North America. June 1999; 46: Issue 3. 2. Bryan McIver, Susan A Romanski; Todd B Nippoldt. Evaluation and Management of Amenorrhea. Mayo Clinic Proceedings Dec 1997; 72: 1161-1169. 3. Tarannun Master Hunter, Diana L.Heiman. Amenorrhea: Evaluation and Treatment. Am Fam Physician 2006; 73: 1374-1382. 4. The Practice Committee of the American Society for Reproductive Medicine. Current Evaluation of Amenorrhea. Fertility and Sterility Nov 2006: 86 Supl 4: 148-155.

37.- De las siguientes infecciones por virus, Cul es la que se asocia y se piensa que puede ser el probable origen de la neoplasia intraepitelial cervical (NIC)?

a) b) c) d)

Herpes tipo 1. Herpes tipo 2. Citomegalovirus. Virus del papiloma humano.

El virus del papiloma humano (HPV) es un virus ADN, con epiteliotrofismo y ha sido estudiado exhaustivamente enestas dos dcadas por mtodos sofisticados como los de biologa molecular.1 Ferraina divide a los virus ADN oncognicos en tres grupos, los papovavirus incluyendo al HPV, los adenovirus y los herpesvirus.2 De los grupos virales mencionados es tema de nuestra investigacin el HPV. Los virus del papiloma humano infectanselectivamente el epitelio de la piel y las mucosas. Estas infecciones pueden ser asintomticas, producir verrugas oasociarse a diversas neoplasias, benignas y malignas.3 El virus del papiloma humano presenta ms de 73 tipos bien definidos; entre ellos hay tipos que comnmente serelacionan con lesiones benignas y otros bien reconocidos que implican un alto grado de transformacin maligna. Los tipos diferentes comparten menos del 90% de sus secuencias de ADN en L1.3 Los HPV son especficos de especiey los tipos concretos se asocian a manifestaciones clnicas especficas. Reguezi menciona especificidad de sitio de tejidoo latencia del HPV en algunas lesiones epiteliales bucales que se asocian con el HPV. Adems los diferentes tipos de HPV no solo se diferencian por la morfologa de las lesiones que producen, sino tambin por su posible potencial oncognico.4 Es por ello que se los ha clasificado en tres grupos de acuerdo a su potencialoncognico y al tipo de lesin que producen5 basndose en neoplasias intraepiteliales cervicales. Bonnez y variosautores los agrupan en los de mayor riesgo HPV tipos 16 y 18, los de riesgo intermedio HPV tipos 31, 33, 35, 39, 45, 51y 52 y los de menor riesgo los tipos 6, 11, 42 y otros. Serman F: Cncer CU. Epidemiologa, historia natural y relacin del virus papiloma humano. Rev Chil Obstet Ginecol 2002; 67(4): 318-23.

38.- Paciente de 40 aos de edad con miomatosis uterina sintomtica, sin otros patolgicos sistmicos de importancia, con antecedente de cirugas plvicas previas, Cul es el tratamiento de eleccin en esta paciente?

a) b) c) d)

Histerectoma total abdominal. Histerectoma vaginal. Progesterona Anlogos de GnRH.

Eleccin de procedimiento: edad, deseos de fertilidad o de conservar el tero, nmero, tamao y localizacin. Historia y exploracin clnicas. Exploraciones complementarias. Experiencia tcnica. Coexistencia de otros procesos e indicaciones quirrgicas. Valoracin del riesgo quirrgico de las diferentes vas de abordajes. Consentimiento informado de la paciente. Bases generales preciruga:

Adecuada planificacin: Ecografa, RNM, HSG. Consentimiento informado. Correccin de la anemia. Profilaxis antibitica.

HISTERECTOMA Para pacientes post menopusica, con paridad satisfecha o sin deseo de preservar el tero. Criterios del ACOG para Histerectoma por miomas. Presencia de 1, 2, o 3 Miomas asintomticos de tamao tal que son palpables por va abdominal y preocupan a la paciente. Hemorragia uterina excesiva.

Duracin mayor de 8 das. Anemia por prdida sangunea aguda o crnica. Molestias plvicas producidas por los miomas: signos compresivos.

Contraindicaciones: Deseo de conservar la fecundidad. Miomas asintomticos.

3. Contraindicacin mdica o dificultades tcnicas para la ciruga. Referencia: -Guarnaccia M. and Rein M. Traditional Surgical Approaches to Uterine Fibroids Abdominal. Myomectomy and Hysterectomy. Clinical Obstetrics and Gynecology 2001. 44.2. 385-400 - Milad. M and Sankpal R. Laparoscopic Approaches to Uterine Leiomyomas. Clinical Obstetrics and Gynecology. 2001. 44-2. 401-411 - Carlson K. et al. Indications for Hysterectomy. N. Engl. J. Med. 1993. 328(12) 56-

39.- En una paciente con amenorrea secundaria que presenta menstruacin posterior a la administracin de progestgenos, lo ms probable es que tenga? a) b) c) d) Estrgenos bajos Estrgenos normales Progesterona elevada Gonadotropinas altas

La presencia de estrgenos se puede establecer de dos maneras: con la prueba de desafo con acetato de medroxiprogesterona (AMP) y el ndice de maduracin vaginal. Si hay estrgenos, luego de la administracin de 10 mg de AMP diarios por 10 das debera presentarse un sangrado por vagina. Y de esta forma corroborar los niveles de estrgenos serian normales. La ausencia de esta metrorragia sugiere una obstruccin o un hipogonadismo. La otra alternativa es realizar el ndice de madurez vaginal que es un procedimiento sencillo que puede obtenerse en el consultorio. Usando una esptula como las de Papanicolaou el gineclogo extrae una muestra de la vagina y la extiende en un portaobjeto. Este portaobjeto se procesa y se estudia en el microscopio. Un extendido maduro tendr clulas epiteliales superficiales grandes y numerosas con un ncleo rodeado por una gran cantidad de citoplasma. Por el contrario, una muestra no estrognica tendr un mayor nmero de clulas parabasales y basales con ncleo grande rodeado por escaso citoplasma. Referencia: Speroff L, Glass RH, Kase NG, eds. Clinical Gynecologic Endocrinology and Infertility, 5th ed. Baltimore: Williams & Wilkins; 1994:334-335.

40.- Es el sndrome que ms comnmente es causa de amenorrea primaria: a) b) c) d) Turner Feminizacin testicular Rokitansky Asherman

Amenorreas primarias Se considera amenorrea primaria a todos los casos en que la menarquia no haya tenido lugar antes de los 16 aos (el 97% de las mujeres tienen la menarquia antes de los 15 aos y medio) o antes de los 14 aos si no tiene caracteres sexuales secundarios .La causa ms frecuente de amenorrea primaria es la disgenesia gonadal (defectuosa formacin de los ovarios). Las principales causas de amenorrea primaria incluyen el sndrome de Turner, el sndrome de insensibilidad andrognica y la agenesia tero-vaginal. Para el diagnstico, es prctico relacionar este sntoma con el grado de desarrollo puberal, la talla y con la presencia o no

de anormalidades en el tracto genital y es til la siguiente clasificacin, que ser revisada desde el punto de vista gentico: A) Amenorrea con retraso puberal: Hipogonadismo hipergonadotrpico: sndrome de Turner (ST), sndrome de Swyer (XYGD), sndrome de Perrault y otras formas de disgenesia gonadal XX (XXGD). Hipogonadismo hipogonadotrpico: sndrome de Kallmann (KS) B) Amenorrea con pubertad normal: Sndrome de ovario poliqustico (SOP) Hiperplasia adrenal congnita no clsica Sndrome de Mayer-Rokitansky-Kster-Hauser (MRKH) Sndrome de insensibilidad a los andrgenos (SIA) referenciaS biblioGrficaS 1. ASRM. Practice Committee of the American Society for Reproductive Medicine. Current evaluation of amenorrhea. Fertil Steril. 2004;82 Suppl 1:S33-9. 2. Pletcher JR, Slap GB. Menstrual disorders. Amenorrhea. Pediatr Clin North Am. 1999;46(3):505-18. 3. Iglesias EA, Coupey SM. Menstrual cycle abnormalities: diagnosis and management. Adolesc Med. 1999;10(2):255-73. 4. Aloi JA. Evaluation of amenorrhea. Compr Ther. 1995;21(10):575-8. . 5. Mazzanti L, Cicognani A, Baldazzi L, Bergamaschi R, Scarano E, Strocci S, Nicoletti A, E. Gonadoblastoma in Turner syndrome and Y-chromosome-derived material. Am J Med Genet. 2005;135(2):150-4.

41.- Efecto colateral comn de los anticonceptivos hormonales de dosis baja: a) b) c) d) Aumentan el riesgo de cncer mamario Aumentan el riesgo de cncer endometrial Tienen alta probabilidad de causar tromboflebitis Producen frecuentemente sangrados intermenstruales

Sangramiento intermenstrual: Es uno de los efectos ms comunes, cuando se utiliza hormonal de baja dosis y cuando ocurre debe investigarse lo siguiente: si no se ingiri la pldora a la misma hora, si se olvid su ingestin durante un da, si se tomaron otros frmacos concomitantes, as como disminucin de la absorcin. Cuando ocurre el comienzo de su uso, ceder sin ningn otro tipo de tratamiento en el 50 % de las pacientes. En la actualidad no se considera conveniente duplicar la dosis de las pldoras para controlar el sangramiento, pues se incrementa el impacto atrfico en el endometrio por el estmulo progestacional. Referencia: 1. Botella Llusi J, Clavero Nez JA. Contracepcin. En: Tratado de ginecologa. 14 ed.

42.- Paciente de 34 aos con 39 SDG; se reportan cifras de TA 145/95, proteinuria 2+, reflejos patelares hiperactivos. El diagnstico ms probable es:

a) Glomerulonefritis aguda b) Hipertensin esencial c) feocromocitoma d) Preeclampsia

La hipertensin es la complicacin mdica ms comn del embarazo , aunque para algunos autores es la segunda complicacin mdica del embarazo slo despus de la anemia; es ms frecuente en jvenes durante el primer embarazo y en nulparas de mayor edad, hipertensas previas y diabticas. En Mxico, tambin es la complicacin ms frecuente del embarazo, la incidencia es de 47.3 por cada 1 000 nacimientos y es adems, la primera causa de ingreso de pacientes embarazadas a las unidades de terapia intensiva (debido a hemorragia masiva, para recibir soporte hemodinmico), segn la secretara de salud (2001) la mortalidad por complicaciones del embarazo ocupa el 15 lugar en la mortalidad hospitalaria en general. Adems, la tasa de preeclampsia se ha incrementado 40% en el periodo entre 1990 y 1999 y constituye hasta 40% de los partos prematuros iatrognicos.
.

Preeclampsia La preeclampsia es un sndrome clnico caracterizado por hipertensin con disfuncin orgnica mltiple, proteinuria, edemas. Es definida como un incremento de al menos 140/90 mmHg despus de la semana 20 de gestacin, un incremento en la presin sangunea diastlica de al menos 15 mmHg respecto a un nivel previo a la semana 20 combinado con proteinuria (> 300 mg en 24 horas). Las mediciones de la presin arterial citadas deben ser medidas al menos 2 ocasiones con por lo menos 6 horas de separacin. La proteinuria puede ser una toma simple de orina al azar que indique al menos 30 mg/dL 3 ++ en dos muestras de orina1 segn el tipo de prueba. El criterio del incremento de 30 mmHg en la presin sistlica y/o 15 mmHg en la presin diastlica respecto a valores previos a la semana 20 de gestacin ha sido eliminado por ser poco especfico15

1.Myers JE, Baker PN. Hupertensive diseases and eclampsia. Curr Opin Obstet Gynecol 2002; 14: 119-125 2. Tierney, McPhee, Papadakis. Diagnstico clnico y tratamiento 2003. 38 ed, Mxico, Manual Moderno, 2003: 770-773. 3. Wilson MI, Goodwin TM, Pan VI, Ingles SA. Molecular epidemiology of preeclampsia. Obstet and Gynecol Survey 2003; 58(1):39-66.

4. Burrow GM. Complicaciones mdicas durante el embarazo. 4 ed, Mxico, McGraw-Hill panamericana: 1996: 1-25. 5. Guyton AC, Hall JE. Embarazo y lactancia en: Tratado de fisiologa mdica, 10 ed, Mxico, McGraw-Hill Interamericana 2001: 1135-45. 6. Vaticon D. Fisiologa de la fecundacin, embarazo, parto y lactancia, en: Tresguerres JAF. Fisiologa Humana. Mxico, Interamericana McGraw-Hill, 1992: 1086-110. 7. Pridjian G, Puschett JB. Preeclampisa. Part 1: Clinical and Pathophysiologic Considerations. Obstet and Gynecol Survey 2002; 57 (9): 598-618. 8. Pridjian G, Puschett JB. Preeclampisa. Part I1: Experimental and Genetic Considerations. Obstet and Gynecol Survey 2002; 57 (9): 619-40. 9. IMSS. Embarazo de alto riesgo. Gua diagnstica teraputica. Rev Med IMSS 1998; 36(1):45-60.

43.- Una mujer embarazada, puede afectar al feto y hacerlo contraer lesiones importantes durante el embarazo o al salir al exterior (atravesando el canal de parto), s la gestante se encuentra afectada de la siguiente patologa:

a) Herpes genital b) Gardenerella c) Tricomonas d) Gonococos

Herpes genital La prevalencia de herpes simplex genital o tipo 2 (VHS-2) en mujeres embarazadas vara entre 7 y 33% en distintas series. La prevalencia ha experimentado un sostenido aumento durante los ltimos aos. Se estima que aproximadamente 1 a 3% de las mujeres adquiere cada ao la infeccin. En el caso de parejas discordantes, la tasa de adquisicin aumenta hasta 10 a 30% anual. La adquisicin durante el embarazo es ~2%. La transmisin al hijo ocurre principalmente cuando la mujer embarazada adquiere una infeccin primaria. La transmisin es de 30 a 50% cuando la infeccin primaria ocurre cerca del momento del parto. La mayor transmisin (85%) ocurre durante el parto. Sin embargo, tambin puede ocurrir transmisin intrauterina (5-8%) y post-natal (8-10%). Los factores que inciden en la transmisin son: infeccin primaria mucho mayor eficiencia que

infeccin recurrente, parejas discordantes, ttulos de anticuerpos maternos procedimientos obsttricos invasores, (los que estn absolutamente contraindicados).

Las manifestaciones en la mujer embarazada son principalmente bajo la forma de herpes genital localizado, muy raramente ocurre diseminacin cutnea y visceral, situacin de elevada mortalidad (50%). La infeccin en el nio, si ocurre en las primeras 20 semanas del embarazo, puede provocar aborto en 25%, malformaciones cerebrales, cicatrices, corioretinitis, RCIU. Si ocurre despus de las 20 semanas, puede causar parto prematuro, RCIU, o herpes neonatal. Esta condicin clnica tiene tres formas de presentacin, las dos primeras de elevada mortalidad y secuelas: herpes diseminado y encefalitis herptica o infeccin localizada en piel, ojo y boca.

Referencia: Pass R, Weber T, Whitley RJ. Herpesvirus infections in pregnancy. Recommendations from the International Herpes Management Forum. Management Strategies Workshop and 7th Annual Meeting. Whitley R J. Varicella - Zoster virus. Mandell, Douglas and Bennett's Principles and Practice of Infectious Diseases. Mandell G, Bennett J, Dolin R, editors. Fifth edition, 2000 Churchill Livingstone, Philadelphia, pp: 1586-98. Abarca K, Cohen B J, Vial P. Seroprevalence of parvovirus B19 in urban Chilean children and young adults, 1990 and 1996. Epidemiol Infect 2002; 128: 5962.

44.- La forma mas frecuente de tumor de cuello uterino es: a) Condiloma acuminado. b) Mioma. c) Plipo endocervical. d) Tumor mesonefrico.

Plipo endocervical Es una hiperplasia focal de la mucosa. El plipo es generalmente pediculado, con mayor frecuencia se halla en el canal. Generalmente tiene un eje conjuntivo vascularizado e inflamado. La mayora de los plipos son crecimientos anormales benignos, que se forman en el borde de crvix (cervicales), en el propio canal cervical (endocervicales) o dentro del tero (endometriales).

La mayora de los plipos endocervicales tiene forma pediculada o de lgrima, es decir, que estn unidos al tejido por una base muy finita y su masa total cuelga como una lgrima. Pueden salir a cualquier altura del canal cervical y crecer hasta asomar en la vagina. Algunas veces son detectados porque causan sangrado entre menstruaciones y eso da la voz de alerta, o porque resultan molestos durante las relaciones sexuales.

Referencia: 1. Kurman RJ. Blausteins pathology of the female genital tract. 4ed. New York: Springer Verlag, p. 394-8, 1994.

45.- Las hiper y polimenorreas o prdidas de sangre continuas sin conservacin del ciclo, son ms frecuentes en los miomas de localizacin:

a) Submucoso b) Intramural d) Intraligamentaria e) cervical

Descripcin:
Los miomas son tumores monoclonales benignos de las clulas del msculo liso del miometrio. Estn compuestos por grandes cantidades de matriz extracelular que contiene colgeno, fibronectina y proteoglicanos. El colgeno tipo I y tipo II es abundante, pero las fibrillas de colgeno son anormales y estn desorganizadas, de modo similar a lo que se observa en la formacin de queloides. Los miomas submucosos son los menos frecuentes, constituyendo nicamente el 5% de la totalidad de los miomas, pero tambin son los ms asintomticos, ya que muy a menudo producen un aumento del sangrado menstrual en forma de hiper y polimenorreas, e incluso hemorragias importantes que exigen tratamiento de urgencia. Las metrorragias tambin son habituales en este tipo de miomas. Segn Novak, el peligro de degeneracin sarcomatosa es mucho mayor en los miomas submucosos, e igualmente es causa de dismenorreas ms intensas y frecuentes.

BIBLIOGRAFA 1. De la Fuente U. Tratado de Obstetricia y Ginecologa. Mc Graw-Hill. (Madrid). 1998. Volumen II. 2. DI SAIA S. Tratado de Obstetricia y Ginecologa de Danforth. Sexta. Mc Graw-Hill. (Nueva York). 1990. Sexta Edicin. 3. Gonzlez-Merlo J. Tratado de Obstetricia y Ginecologa. Salvat Editores S.A. (Barcelona). 1990. Quinta Edicin. 4. Aller J., Pages G. Obstetricia Moderna. Tercera Edicin. Mc Graw-Hill. (Caracas). 1999. 5. Formacin Mdica Continuada en Atencin Primaria. Marzo 1995. Vo. 5, N (3).

46.- Mujer de 24 aos acude al servicio de ginecologa, por referir ciclos opso-menorreicos desde el inicio de su menarquia, en los ltimos 7 das ha incrementado 15 Kg. de lo que pesaba habitualmente, se aprecia una gran cantidad de acne, pero adems refiere depilarse el rea del bigote cada semana, y cree que esto le sensibiliza la piel para que aumente el acne. En el caso de ovario poliqustico el dato clnico que con ms frecuencia les acompaa es:

a) b) c) d)

Anovulacin y esterilidad. Hirsutismo. Amenorrea. Obesidad.

El sndrome de ovarios poliqusticos (SOPQ) afecta aproximadamente a un 4% de mujeres en edad reproductiva y se caracteriza por anovulacin crnica e hiperandrogenismo. Es la causa ms comn de infertilidad en mujeres. Se caracteriza clnicamente por acn, alopecia, hirsutismo, irregularidades menstruales e infertilidad. Los hallazgos de laboratorio ms frecuentes son: aumento de la hormona luteinizante (LH), aumento de la relacin LH/FSH (hormona folculoestimulante), aumento de andrgenos (tanto ovricos como adrenales) y de estrgenos circulantes. Otros hallazgos de laboratorio habituales son una prueba tolerancia oral a la glucosa anormal y alteraciones en el perfil lipdico. Todo esto junto con las imgenes ecocardiogrficas caractersticas define al sndrome. La teraputica permite dos grandes enfoques que pueden superponerse: la correccin de las manifestaciones de hiperandrogenismo y el tratamiento de las alteraciones del eje reproductivo (anovulacin, esterilidad). Los antiandrgenos estn fundamentalmente indicados para tratar los sntomas virilizantes. Las alternativas para inducir la ovulacin son numerosas: al citrato de clomifeno y a la antigua reseccin en cua se agregan las gonadotrofinas humanas, pulsos de GnRH (hormona liberadora de gonadotrofinas), medidas o frmacos para modificar los niveles de insulina, y finalmente tcnicas quirrgicas endoscpicas para reducir la masa ovrica. BIBLIOGRAFIA 1. Guzick D.Polycystic ovary syndrome: Symptomatology, pathophysiology, and epidemiology. Am J Ostetric Gynecol 1998; 179 (6): 89-93. 2. Stephen Franks. Polycystic ovary syndrome. N Engl J Med 1995; 333(13): 853-861. 3. Gori J.R., Larusso A. Ginecologa de Gori. 2 Edicin. Buenos Aires, Argentina. Editorial El Ateneo. 2001. 4. Adams J., Polson D. W., Franks S. Prevalence of polycystic ovaries in women with anovulation and idiopathic hirsutism. Br Med J 1986; 293: 355-9. 5. Copeland L. J . Ginecologa. Buenos Aires, Argentina. Editorial Panamericana. 1 Edicin. 1994. 6. Ehrmann D.A., Rosenfield R.L., Barnes R.B., Brigell D.F., Sheikh Z. Detection of functional ovarian hyperandrogenism in women with androgen excess. N Engl J Med 1992; 327:157-162. 7. Kahasar-Miller M., Conway Myers B., Boots L., Azziz R. Steroidogenic acute regulatory protein (StAR) in the ovaries of healthy women and those with polycystic ovary syndrome. Am J Obstet Gynecol 2001; 185(6): 1381-7. 8. Prez Snchez A. Ginecologa. Santiago de Chile. Publicaciones Tcnicas Mediterrneo. 3 Edicin. 1995. 9. Velzquez E., Mendoza S., Hamer T., Sosa F., Glucck C. Metformin therapy in women with polycistic ovary syndrome reduces hiperinsulinemia, insulin resistence, hyperandrogenemia, and systolic blood pressure, while facilitating menstrual regularity and pregnancy. Metabolism 1994 ; 43: 647-655.

47.-Femenino de 26 aos con abortos espontneos recurrentes, Cul es el factor etiolgico ms comn de aborto espontneo del primer trimestre?

a) b) c) d)

Anormalidades cromosmicas del embrin. Anormalidades hormonales de la madre. Malformaciones uterinas. Enfermedades sistmicas de la madre.

Es el embarazo que termina antes de las 20 semanas de gestacin, durante el aborto el feto, la placenta y el saco de lquido alrededor del feto son expulsados del tero. El aborto se presenta entre un 15-20% de todos los embarazos, el riesgo disminuye conforme el embarazo progrese, es posible que tanto como el 50% de los abortos ocurren antes de la implantacin en el tero, despus de la implantacin la tasa de prdida es de 30% (esto es antes de que sea reconocido clnicamente), despus de ser reconocido es de 25% y disminuye dramticamente despus de las 8 semanas. Muchas son las causas de abortos las ms comunes son las anomalas cromosmicas, estas cuentan con el 50% de los casos que se producen en el primer trimestre, si el aborto se produce ms tarde hay tambin una tasa alta de anomalas cromosmicas, 30% en el segundo trimestre y 5% en el tercer trimestre. La mayora de las anomalas cromosmicas encontradas en los abortos espontneos son las trisomas autosmicas (50% de los especmenes), en el cariotipo la que se ve con ms frecuencia en el abortos es 45 XO, la gran mayora de esas anormalidades genticas son fenmenos espontneos aislados. Se conoce bien la relacin entre la edad materna y la anormalidad cromosmica, hay un aumento claro en la frecuencia de trisomas principalmente 13, 14, 15, 21 y 22 debido a la edad materna, muchos de esos embarazos terminan en abortos temprano y contribuyen, en forma importante al ndice elevado de interrupciones espontneas en esa poblacin de pacientes. Laing FC, Frapes MC. Evaluacin ecogrca durante el primer trimestre del embarazo. En: Callen PW. Ecografa en Obstetricia y Ginecologa. 4a. edicin. Buenos Aires: Editorial Mdica Panamericana; 2002. p. 126-34. 5. Lomax B, Tang S, Separovic E, Phillips D, Hillard E, Thomson T, et al. Comparative genomic hybridization in combination with ow cytometry improves results of cytogenetic analysis of spontaneous abortions. Am J Hum Genet 2000;66:1516-21.

48.- Femenino que cursa con 36. 5 semanas de gestacin acude al servicio por referir malestar general, fosfenos, nausea y vmito, aprecia moderada ictericia, usted sospecha de un sndrome de HELLP Que alteraciones de laboratorio espera encontrar al confirmar el diagnstico?: a) b) c) d) Anemia Hemoltica, trombocitopenia, enzimas hepticas elevadas. Trombocitosis, Enzimas heptica elevadas, Anemia hemoltica. Anemia hemolitica, Trombocitosis, Fosfatasa Alcalina elevada. Trombocitopenia, Leucopenia, Hipertensin Arterial.

DEFINICIN: Es una complicacin de la preeclampsia en la cual adems de la Hipertensin Arterial y proteinuria hay presencia de anemia hemoltica, enzimas hepticas elevadas y recuento bajo de plaquetas

EPIDEMIOLOGIA: Se presenta en un 4 a 10% de las preeclmpticas, diagnosticndose anteparto en un 70% de los casos preferentemente antes de las 37 semanas, mientras que el 30% de los casos restantes enferma en los primeros 7 das del puerperio, sobre todo en las 48 h iniciales. La proteinuria e hipertensin pueden estar ausentes en un 15 al 20% de los casos. Incidencia mayor en multigestantes y en edades avanzadas. Ocurre ms frecuentemente cuando se demora la salida del feto y cuando se presenta desprendimiento de la placenta Mortalidad materna del 24% y mortalidad perinatal del 30-40%.

CLASIFICACION: Sndrome de HELLP. Clasificacin de Mississipi. CLASE Plaquetopenia LDH 1 Severa <50000 >600 IU/L 2 Moderada >600 IU/L >50000 <100000 3 Ligera >100000 >600 IU/L <150000 PE severa >150000 <400 IU/L Eclampsia (sin HELLP)

AST-ALT >70 IU/L >70 IU/L

>40 IU/L <70 IU/L <40IU/L

MANIFESTACIONES CLINICAS: Malestar general, fatiga y molestias inespecficas 90% Cefalea 70% Epigastralgia 64% Vmito 22% Fosfenos 15% Visin Borrosa 11% Acfenos 3% Ictericia Anemia no explicada Oliguria

Si se aade una HEMORRAGIA HEPTICA, el paciente puede quejarse de dolor en el HOMBRO DERECHO y EL CUELLO, adems de las molestias abdominales. Equimosis en los sitios de punciones venosas, petequias en los sitios de presin del brazo, pero pueden tener pruebas de Rumpel Leed negativas. En casos severos se pude presentar ascitis como causa de hipertensin portal.

DIAGNOSTICO: El diagnstico clnico del sndrome de HELLP se plantea en gestantes o purperas con preeclampsia severa-eclampsia, excepto en el 15-20%, en las cuales esta asociacin no puede ser demostrada, en tanto se cumplan los criterios de Sibai:

MANIFESTACIONES CLINICAS: Malestar general, fatiga y molestias inespecficas 90% Cefalea 70% Epigastralgia 64% Vmito 22% Fosfenos 15% Visin Borrosa 11% Acfenos 3% Ictericia. Anemia no explicada. Oliguria.

Si se aade una HEMORRAGIA HEPTICA, el paciente puede quejarse de dolor en el HOMBRO DERECHO y EL CUELLO, adems de las molestias abdominales. Equimosis en los sitios de punciones venosas, petequias en los sitios de presin del brazo, pero pueden tener pruebas de Rumpel Leed negativas. En casos severos se pude presentar ascitis como causa de hipertensin portal.

DIAGNOSTICO: El diagnstico clnico del sndrome de HELLP se plantea en gestantes o purperas con preeclampsia severa-eclampsia, excepto en el 15-20%, en las cuales esta asociacin no puede ser demostrada, en tanto se cumplan los criterios de Sibai:

HEMOLISIS Frotis perifrico anormal (eritrocitos fragmentados) Hematocrito (>24%) Bilirrubina indirecta (>1.2mg/dL) Deshidrogenasa lctica (>218 UI/L)

ENZIMAS HEPTICAS ELEVADAS

LDH >218UI/L AST >30UI/L ALT >37UI/L

PLAQUETAS BAJAS <100.000/mm3

BIBLIOGRAFIA: 1. Sibai baha, El sndrome HELLP. Universidad de Valencia , revista quincenal de Obstetricia clnica y ginecologa, Octubre 2003. 2. V. Cararach, Sndrome de HELLP y Repercusiones maternas. X curso intensivo de formacin continuada materno fetal. Enero de 2003.

3. Toirac, Abelardo. Sndrome de Weistein HELLP Hospital Ginecoobstetrico Tamara Bunke. Junio 2002 4. De la Fuente, David. Sndrome HELLP. Medicina Universitria 2003; 5 (19): 101 -9 5. Andrea G. Witlin, DO, Baha M. Sibai, MD. Diagnosis and Management of women with Hemolysis Elevate Liver Enzymes, and Pletelet Count (HELLP) syndrome. Hospital Physician. Febrero 1999. 6. CIFUENTES B, Rodrigo. Ginecologa y obstetricia basadas en las evidencias. Bogot: Distribuna, 2006. Sexta edicin. 447 - 283 p.

49.- Es un signo de presuncin de embarazo:

a) Amenorrea. b) Signo de Chadwick. c) Prueba inmunolgica de embarazo positiva. d) Oscurecimiento de areolas y de cicatrices antiguas.

Sntomas y Signos de Presuncin: Son aquellos referidos por la gestante. Existen mltiples sntomas de presuncin del embarazo, de los cuales el ms frecuente es la amenorrea. La ausencia de un perodo menstrual suele ser el primer indicio de un posible embarazo, (si bien se hace mencin a este sntoma como de presuncin, en rigor corresponde a un signo de probabilidad, ya que corresponde al rea genital ); sin embargo podemos encontrar mujeres con oligomenorreas, las cuales no perciben su embarazo hasta que se presentan otros sntomas tales como nauseas, vmitos, malestar frente a determinados olores, cambios de peso, fatiga, tensin mamaria, frecuencia miccional, estreimiento, cambios del apetito, pezones ms oscuros, aumento del volumen abdominal. Ya que estos sntomas no son especficos para embarazo y pueden responder a mltiples cuadros clnicos, para confirmar la presencia de un embarazo deben recurrirse a otros signos.

Referencia Bibliogrfica. 1.- Tratado de Ginecologa, Obstetricia y Medicina Reproductiva. Luis Cabero Roura. 2003. 2.- Obstetricia Williams. 2002 . 3.- Obstetricia Schwarcz 1998.

50.- Una de las siguientes vacunas est indicada y se puede administrar embarazo A cul nos referimos?

durante el

1) Sarampin. 2) Poliomielitis. 3) Ttanos. 4) Rubola.

Vacunas contraindicadas durante el embarazo

SRP ( Triple vrica ). Rubela. Fiebre tifoidea oral.

Las vacunas con virus vivos atenuados no se administran a mujeres embarazadas o con posibilidad de embarazo en los tres meses siguientes. Considerando que el padecer sarampin durante el embarazo aumenta el riesgo de parto prematuro y aborto espontneo una mujer embarazada susceptible que ha estado expuesta al sarampin, segn el ACP, debe recibir Inmunoglobulina intramuscular dentro de los 6 das posteriores a la exposicin y luego del parto debe ser inmunizada con la vacuna MMR por lo menos 3 meses despus de la aplicacin de la inmunoglobulina. El virus de la parotiditis puede infectar la placenta y el feto pero no causa malformaciones congnitas, la inmunoglobulina no es de valor en la profilaxis post exposicin. Cuando la rubola afecta a la mujer embarazada puede producir aborto o muerte fetal o malformaciones congnitas o retraso psicomotor.

Pueden administarse si estn indicadas

Gripe y Neumococo: estn indicadas en mujeres con enfermedades subyacentes de alto riesgo, pero no son de rutina. Hepatitis B: el embarazo no es una contraindicacin para mujeres que pertenecen a grupos de alto riesgo. Se debe realizar de rutina deteccin de antgeno de superficie, si es seronegativa y tiene alto riesgo se vacuna. Hepatitis A: si la exposicin se produjo durante el embarazo, se estudia la presencia de anticuerpos contra hepatitis A, si es negativa se administra inmunoglobulina antes de 1

semana de la exposicin. Si no se puede realizar la deteccin de anticuerpos, se administra la inmunoglobulina. Rabia : de acuerdo al riesgo Poliomielitis: slo cuando el riesgo de exposicin es alto. Para una mujer embarazada no inmune con alto riesgo de exposicin al virus de la polio se recomienda una sola dosis de la vacuna oral si se dispone de menos de 4 semanas y se requiere de proteccin inmediata. Pero si el tiempo lo permite pueden administrarse dos dosis de la vacuna antipolio inactivada con potencia incrementada con intervalos de 1-2 meses; las dosis restantes se administran en intervalos apropiados. Las mujeres con alto riesgo que han completado la primera serie de vacunas ms de 10 aos atrs deben colocarse una dosis de vacuna inactivada una sola vez como refuerzo. Fiebre amarilla: si la mujer embarazada debe viajar a un rea dnde esta patologa es prevalente, se debe vacunar con virus vivo atenuado. Varicela Zster: deben recibir inmunoglobulina especfica contra el virus Varicela Zster, las mujeres embarazadas expuestas y las que adquieren el virus en forma primaria durante el embarazo.

Vacunas indicadas

Ttanos y Difteria: las mujeres que no han sido inmunizadas deben recibir dos dosis de toroide tetnico y diftrico separados por 4 a 8 semanas, en el 2 o 3 trimestre. La tercera dosis, 6 a 12 meses despus. si la primera serie no se complet o si fue inmunizada ms de 10 aos atrs debe administrase una o dos dosis de refuerzo.

Referencia: 1. Centers for Disease Control & Prevention. Prevention of Hepatitis A Through Active or Passive Immunization: Recommendations of the Advisory Committee on Immunization Practices (ACIP). MMWR 48 (No. RR-12): 24, 1999. 2. Centers for Disease Control & Prevention. Hepatitis B Virus: A Comprehensive Strategy for Eliminating Transmission in the United States Through Universal Childhood Vaccination: Recommendations of the Immunization Practices Advisory ommittee (ACIP). MMWR 40 (No. RR-13): 4, 1991. 3. Centers for Disease Control & Prevention. General Recommendations on Immunization: Recommendations of the Advisory Committee on Immunization Practices (ACIP). MMWR 51 (No. RR-2): 18-19, 2002.

51.En la sala de urgencias recibe a una paciente que inicia con convulsiones por preclampsia usted decide administrar el siguiente frmaco ya que es el de eleccin en sta patologa:

a) Sulfato de magnesio. b) Diacepam. c) Fenitona. d) Donadores de xido ntrico.

Manejo de la Preeclampsia 1. Manejo ambulatorio: HTA sin proteinuria significativa, se recomienda el reposo en cama. Monitoreo de TA, peso, presencia de protenas en orina. Ecografas peridicas para ver el feto y evaluar posibles retardo de crecimiento. 2. Manejo hospitalario: para mujeres con HTA inducida por el embarazo y 2+ o ms o proteinuria significativa y en quienes fall el manejo ambulatorio. 3. Laboratorio y evaluacin del peso: debe realizarse diariamente. Evaluacin de la dinmica fetal. Monitoreo de sntomas como cefalea, alteraciones visuales y dolor epigstrico. 4. El parto es el tratamiento de eleccin: el cual debe realizarse cuando el feto est maduro pero puede realizarse en forma temprana si la salud de la madre est en peligro o si hay evidencia de distress fetal. El parto est indicado cuando la paciente cumple con los criterios de preeclampsia severa. Betametasona 12.5 mg IM dos veces por da puede estimular la maduracin de los pulmones fetales. 5. Terapia antihipertensiva: est indicada slo si la TA es persistentemente > 160/110 , es importante disminuir la TA hasta una diastlica de 90 a 100 mmHg porque la presin normal podra resultar en hipoperfusin de la placenta. Los diurticos nunca estn indicados, estas pacientes ya son hipovolmicas. Los IECA no deben ser usados durante el embarazo. Las medicaciones de largo plazo, incluyen alfa metildopa, atenolol y labetalol. 6. Terapia anticonvulsivante: A- Profilaxis de las convulsiones: est indicada en todas las pacientes pre-eclmpticas durante el trabajo de parto y el parto y por un mnimo de 24 hs luego del mismo. Algunos mantienen la terapia con magnesio hasta que comienza la diuresis. El Sulfato de Magnesio es la droga de eleccin. La dosis profilctica es de 4 a 6 g de sulfato de magnesio IV y contina con 2 g c/ hora. B- Tratamiento de las convulsiones: Sulfato de Magnesio 1 g/min IV hasta controlar las convulsiones hasta un mximo de 4 a 6 g. El nivel teraputico es de 4 meq/l. Toxicidad del magnesio: ausencia de reflejo patelar, debilidad muscular, parlisis respiratoria y depresin cardaca, 10 ml al 10 % de gluconato de calcio puede ser administrada IV. La terapia con sulfato de magnesio contina por lo menos 24 horas en el post parto, la terapia puede detenerse si la excrecin urinaria es > 200 ml/h por cuatro horas consecutivas. C- Prevencin: 81 mg de aspirina diarios pueden ser administrados luego del primer trimestre en mujeres con hipertensin crnica o historia previa de preeclampsia, sin embargo la eficacia de esta indicacin ha sido cuestionada. Myers JE, Baker PN. Hupertensive diseases and eclampsia. Curr Opin Obstet Gynecol 2002; 14: 119-125 2. Tierney, McPhee, Papadakis. Diagnstico clnico y tratamiento 2003. 38 ed, Mxico,

Manual Moderno, 2003: 770-773. 3. Wilson MI, Goodwin TM, Pan VI, Ingles SA. Molecular epidemiology of preeclampsia. Obstet and Gynecol Survey 2003; 58(1):39-66. 4. Burrow GM. Complicaciones mdicas durante el embarazo. 4 ed, Mxico, McGraw-Hill panamericana: 1996: 1-25. 5. Guyton AC, Hall JE. Embarazo y lactancia en: Tratado de fisiologa mdica, 10 ed, Mxico, McGraw-Hill Interamericana 2001: 1135-45. 6. Vaticon D. Fisiologa de la fecundacin, embarazo, parto y lactancia, en: Tresguerres JAF. Fisiologa Humana. Mxico, Interamericana McGraw-Hill, 1992: 1086-1109. 7. Pridjian G, Puschett JB. Preeclampisa. Part 1: Clinical and Pathophysiologic Considerations. Obstet and Gynecol Survey 2002; 57 (9): 598-618. 8. Pridjian G, Puschett JB. Preeclampisa. Part I1: Experimental and Genetic Considerations. Obstet and Gynecol Survey 2002; 57 (9): 619-40. 9. IMSS. Embarazo de alto riesgo. Gua diagnstica teraputica. Rev Med IMSS 1998; 36(1):45-60.

52.- Si la placenta se inserta e infiltra, la serosa uterina se denomina? a) b) c) d) Placenta Percreta. Placenta Increta. Placenta Acreta. Placenta Percreta dextruens.

La diferenciacin entre el acretismo y el percretismo placentario esanatomopatolgica y clnica. Desde el punto de vista histolgico, ambasentidades se diferencian por le grado de invasin placentaria en el miometrio.Pero ms importante aun, es el diagnstico clnico, este nos indicar realmenteuna actitud activa frente a este padecimiento. El primer diagnstico sin duda, lo constituye la sospecha clnica. Todoobstetra que se encuentre hoy con una paciente con placenta previa y cesreasanteriores, debe buscar perfeccionar y dirigir los mtodos de diagnstico paraconfirmarlo o descartarlo. Dentro de los mtodos de diagnstico, el ultrasonido es el ms habitual, elms econmico y el ms difundido. Sin embargo, en ocasiones la informacinanatmica puede no ser concluyente, para ellos la RMN con contraste brindainformacin crucial. Esta informacin acerca de la vascularizacin(Angiorresonancia), de los planos anatmicos y del grado de invasin, modificala estrategia quirrgica. Referencia: Palacios J, Jos Tratamiento conservador uterino en trastornos adherencialesde la placenta (acretismo y percretismo placentario)Rev ChilObstet Ginecol 1998; 5: 363369

53.- Antiepilptico que administrado en mujeres embarazadas puede producir en el recin nacido, malformaciones y una deficiencia de factores de la coagulacin dependientes de vitamina K: a) Clonazepam. b) Valproato sdico. c) Fenitona. d) Carbamazepina.

Embarazo y lactancia: Al igual que con otros antiepilpticos, existe relacin entre la administracin de fenitona y la aparicin de malformaciones congnitas, por ello no debera utilizarse como frmaco de eleccin durante el embarazo, especialmente durante el primer trimestre, debindose evaluar en cada caso la relacin beneficio/riesgo. No se deber suspender la medicacin antiepilptica si se est administrando para prevenir crisis de gran mal, ya que puede precipitarse un status epilpticus, lo que conlleva el riesgo de producir hipoxia grave en la madre y en el feto. Durante el embarazo puede incrementarse la frecuencia de convulsiones debido a una alteracin en la absorcin o metabolismo de la fenitona. Por ello es muy importante determinar los niveles sricos a fin de adecuar la posologa en cada paciente. Despus del parto probablemente sea necesario reinstaurar la dosis previa al embarazo. La fenitona administrada antes del parto produce dficit de vitamina K, y por tanto de los factores de coagulacin relacionados con esta vitamina. El riesgo de sangrado en la madre o en el recin nacido puede prevenirse o corregirse con la administracin profilctica de esta vitamina a la madre en el ltimo mes de embarazo y al recin nacido inmediatamente despus del nacimiento. No se recomienda la lactancia del neonato por parte de madres tratadas con fenitona, ya que este medicamento se secreta en la leche materna. Referencia: Shepard TH. Catalog of Teratogenic Agents. Johns Hopkins University Press London, 1987. Piper JM. Banur C, Kennedy DI. Prescription drugs use in pregnancy in a Medicaid population. Am J Obstet Gynecol 1987; 157:148-156. Salvador J, Martnez-Fras ML, Rodrguez Pinilla E. Consumo de medicamentos por la mujer embarazada en Espaa. Ministerio de Sanidad y Consumo. Madrid, 1989. Grupo de trabajo DUP. Estudio multicntrico sobre el uso de medicamentos durante el embarazo en Espaa (I). Mtodos y caractersticas demogrficas de la poblacin estudiada. Med Clin 1990;95:764-767. Grupo de trabajo DUP. Estudio multicntrico sobre el uso de medicamentos durante el embarazo en Espaa (II). Los frmacos utilizados durante la gestacin. Med Clin 1991;96:11-15. Grupo de trabajo DUP. Estudio multicntrico sobre el uso de medicamentos durante el embarazo en Espaa (III). Los frmacos utilizados durante el primer trimestre de la gestacin. Med. Clin 1991;96:52-57.

Martnez-Fras ML. Medicamentos y teratogenia. Revisin bibliogrfica y situacin en Espaa. Ministerio de Sanidad y Consumo. Madrid, 1989. Lammer EJ, Chen DT, Hoar RM, et al. Retinoic acid embryopathy. N Engl J Med 1985; 313:837-841.

54.-El rgano del tracto genital que con mayor frecuencia se afecta en la tuberculosis genital es:

1) El ovario. 2) La trompa 3) El endometrio. 4). El crvix. Definimos la tuberculosis genital femenina como la afectacin del aparato genital por el Mycobacterium tuberculosis humano (bacilo cido-alcohol resistente aerobio); excepcionalmente, en mujeres que ingieren leche sin tratar, puede producirse por el tipo bovino. Esta infeccin genital es hoy una curiosa casualidad. Supone en la actualidad aproximadamente el 0,2% de todos los estudios histolgicos de un laboratorio de Anatoma Patolgica. Tanto la incidencia como la prevalencia y riesgo de infeccin anual han disminuido significativamente en pases desarrollados. Ultimamente se seala un aumento de esta incidencia tanto general como en el aparato genital en mujeres con status socio-econmico bajo, drogodependientes y con HIV positivo, de tal modo que en algunas clasificaciones se considera SIDA a aquellas mujeres HIV positivas con tuberculosis asociada. Clasificacin La tuberculosis genital es siempre secundaria a una primoinfeccin pulmonar, que suele ser asintomtica, acompaada de una diseminacin por va hematgena, a troves de los ganglios biliares y el conducto torcico, a todos los rganos de la economa y especialmente a aquellos con mayor PO2, donde crece ms favorablemente. Las trompas pueden tener diferentes grados de afectacin, desde la simple hiperemia, conservando la permeabilidad, a la aparicin de trompas dilatadas, con el extremo distal conglutinado, con paredes fibrosadas y con casium en su interior (forma fibra-caseosa). Histolgicamente se observan abundantes folculos de Koster con crecimientos adenomatosos de las fimbrias y sinequias entre ellas (disposicin en rueda de carro). La trompa suele estar afectada en el cien por cien de los casos. Desde aqu puede diseminarse de forma descendente a endometrio (80%), miometrio (20%), crvix (23%). El ovario se afecta por implantes adherenciales en el 11%. Clnicamente suele manifestarse por dolor en hipogastrio discreto y crnico (evolucin silenciosa), junto a trastornos menstruales (hipomenorrea, amenorrea) y sntomas generales (astenia, anorexia, febrcula, sudoracin nocturna, etc.)

El Manual Merck. Rober Berkow, Andrew J. Fletcher. Editorial Ocano - Centrum, Espaa. Novena Edicin. 1994. Pginas 140 - 157. Principios de Medicina Interna de Harrison. Kurt J. Isselbacher, Eugene Brawnwald, Jean D. Wilson, Joseph B. Martin, Anthony S. Fauci, Dennis L. Kasper. Editorial McGraw -

Hill - Interamericana, Mxico. Dcimotercera edicin. 1994. Volumen I, pginas 827 - 836.

Tratado de Pediatra de Nelson. Richard E. Behrman, Robert M. Kliegman, Ann M. Arvin, Waldo E. Nelson. Editorial McGraw - Hill - Interamericana, Mxico. Dcimoquinta edicin. 1998. Volumen I, pginas 1049 - 1065.

55.- Una paciente de 24 aos, G-1, que cursa con embarazo de 37 semanas de gestacin, presenta prdida del estado de alerta posterior a crisis convulsivas tnico-clnicas, signos vitales con T-A 170.120mmhg Fc 95x, reflejos osteotendinosos aumentados, se aprecia una Fc fetal de 132x y edema importante de miembros inferiores, no se aprecian datos de trabajo de parto ni modificaciones cervicales, El diagnstico ms probable es?

a) Eclampsia. b) Pre eclampsia severa. c) Crisis epileptica de gran mal. d) Hipertensin inducida por el embarazo. CUADRO 1. DIAGNSTICO* Preeclampsia Leve: Se presenta despus de la semana 20 de gestacin, durante el parto, o en las primeras 6 semanas despus de ste Presin sistlica a 140 mm Hg o presin diastlica 90 mm Hg Proteinuria a 300 mg / orina de 24 hrs o su equivalente en tira reactiva Preeclampsia Severa: Se presenta despus de la semana 20 de gestacin, durante el parto, o en las primeras 6 semanas despus de ste. Presin sistlica a 160 mm Hg o presin diastlica 110 mm Hg. Proteinuria a 2 gr en orina de 24 horas o su equivalente en tira reactiva. Creatinina srica > a 1.2 mg/dl. Trombocitopenia 150 000 cel/mm3. Incremento de la deshidrogenasa lctica a 600 UI. Elevacin al doble de los valores de TGO/AST o TGP/ALT. Cefalea, alteraciones visuales o auditivas. Epigastralgia. Oliguria a 500 ml en 24 horas 7. Edema agudo de pulmn. Dolor en hipocondrio derecho. Restriccin en el crecimiento intrauterino. Oligohidramnios.

Eclampsia Preeclampsia mas convulsiones sin otra causa. Se presenta despus de la semana 20 de gestacin, durante el parto, o en las primeras 6 semanas despus de ste. Sndrome de HELLP Criterios para establecer el diagnstico del sndrome de HELLP: Plaquetas < 100 000/mm3 TGO/AST 70U/L DHL 600U/LBilirrubina total > 1.2 mg/dl Se presenta despus de la semana 20 de gestacin, durante el parto, o en las primeras 6 semanas despus de ste. Hipertensin Crnica: Se diagnostica cuando existe hipertensin arterial a 140/90 mm Hg antes de la semana 20 de gestacin o si persiste despus de doce semanas posteriores al parto. Las pacientes con hipertensin crnica deben ser evaluadas antes del embarazo para determinar la severidad de la hipertensin y facilitar la planeacin de un embarazo mediante el cambio de medicamentos y de hbitos higinicos y dietticos para evitar complicaciones. Hipertensin Gestacional: Presencia de hipertensin arterial a 140/90 mm Hg despus de la semana 20 de gestacin y se mantiene hasta las doce semanas despus del parto Ausencia de proteinuria Presencia o no de cefalea, acfenos y fosfenos Despus de 12 semanas de la interrupcin del embarazo se revalorar la presencia de hipertensin, si contina, se reclasifica como hipertensin crnica: es un diagnstico retrospectivo. 8 Si no hay, se clasifica como hipertensin transitoria. Referencia: 1. Aagaard-Tillery KM, Belfort MA. Eclampsia: morbidity, mortality, and management. Clin Obstet Gynecol 48:12-23, 2005. 2. Atallah AN, Hofmeyr GJ, Duley L. Calcium supplementation during pregnancy for preventing hypertensive disorders and related problems. Cochrane Database Syst Rev 1:CD001059, 2001. 3. Barton JR, Sibai BM. Diagnosis and management of hemolysis, elevated liver enzymes, and low platelets syndrome. Clin Perinatol 31:807-33, 2004. 4. Baxter JK, Weinstein L. HELLP syndrome: the state of the art. Obstet Gynecol Surv 59:838-45, 2004. 5. Cetin A. Eclampsia. In Mohler III ER, Townsend RR. Advanced therapy in hypertension and vascular disease. Ontario: B.C. Decker Inc. pp. 407-15, 2006. 6. Cetin A. Hemolysis, elevated liver enzymes, and low platelets (HELLP). In Mohler III ER, Townsend RR. Advanced therapy in hypertension and vascular disease. Ontario: B.C. Decker Inc. pp. 416-20, 2006. 7. Chappell LC, Seed PT, Briley AL, Kelly FJ, Lee R, Hunt BJ, Parmar K, Bewley SJ, Shennan AH, Steer PJ, Poston L. Effect of antioxidants on the occurrence of pre-eclampsia in women at increased risk: a randomised trial. Lancet 354:810-16, 1999.

56. Femenino de 31 aos se envia de alta con diagnstico de enfermedad inflamatoria plvica, regresa a los 15 das con temperatura de 38.5 c, mal estado general y datos de irritacin peritoneal, El diagnstico ms probable es?

a) Endometritis b) Hidrosalpinx c) Absceso tubo-ovrico d) Ooforitis aguda

La enfermedad inflamatoria plvica (EIPA) es un sndrome clnico caracterizado por la infeccin del tracto genital superior que se produce casi siempre por va ascendente desde el cuello uterino. El impacto que la infeccin plvica ejerce sobre la condicin fsica de la mujer va desde la infeccin asintomtica o silente a una mayor morbilidad que en algunos casos puede llegar hasta la muerte. Incluye una variedad de condiciones inflamatorias que afectan el tracto genital superior. Los Centros de Control de Enfermedades (C .D. E.) la definen como un sndrome agudo debido al ascenso de microorganismos de la vagina o el cuello uterino al endometrio, trompas uterinas y en ocasiones a las estructuras vecinas (ovarios, peritoneo y cavidad pelvianas). En el momento actual se incluyen como principales agentes etiolgicos de la E.I.P.A la Neisseria gonorrhedae, las clamydias y los anaerobios. Otros microorganismos como los microplasmas y los actinomices se estn observando con frecuencia. La presencia de anaerobios as como de bacterias aerobias puede deberse a un fenmeno de sobre infeccin secundaria. Hay autores que sealan que excepto para el gonococo y la Clamydia trachormatis, no existen datos suficientes que permitan afirmar que otras bacterias tengan un papel primario en la infeccin de unas trompas sanas. Una vez alterada la integridad anatmica de la trompa, se producira la infeccin mixta o poli microbiana.

El absceso tubo ovrico es una formacin inflamatoria que compromete el ovario y la trompa y puede ser uni o bilateral. En este absceso las estructuras comprometidas estn infectadas y contienen pus. Este proceso inflamatorio es secundario a un proceso infeccioso de la pelvis, habitualmente producido por grmenes muy patgenos, que llegan al tracto genital a travs de una relacin sexual, es decir corresponde a una complicacin severa de una enfermedad de transmisin sexual. Se caracteriza por aumento de volumen del ovario y trompa, los que se encuentran adheridos entre s producto de esta infeccin, adems el proceso infeccioso se extiende habitualmente a otras estructuras y rganos pelvianos, los que estn muy inflamados y adheridos formando lo que se denomina plastrn. El tratamiento se inicia mdicamente con antibiticos de amplio espectro para cubrir tanto grmenes aerbicos como anaerbicos, generalmente requiere de hospitalizacin para iniciar una terapia agresiva endovenosa con los antibiticos y para monitorizar

adecuadamente a la paciente, pues la infeccin general pudiendo llegar hasta la sepsis generalizada. Diagnostico

produce compromiso del estado

El cuadro clnico se sospecha cunado una paciente consulta por dolor abdominal intenso, progresivo, fiebre y compromiso de su estado general, habitualmente en el examen se encuentra un distensin abdominal y a la palpacin del abdomen hay dolor, y signo de blumberg positivo o irritacin peritoneal. El Tacto vaginal demuestra fondos de saco vaginales abombados y dolorosos y habitualmente el cuello del tero lateralizado y doloroso a la movilizacin si el compromiso es unilateral, adems de palpar una masa para uterina irregular y sensible.

BIBLIOGRAFA. 1. 2. 3. 4. 5. 6. 7. 8. Botella Llusi,J.Clavero Nez,J.A:Tratado de Ginecologa.14 edicin.Ed.Diaz de Santos.pg 833-844.Madrid,1993. Brunham,R,C:Infectionin woman and ectopic pregnancy. Am J Obstet Gynecol.67:722,1999. Cates,W,Wasserheit,J,N:Genital Infection Epidemiology and sequeale.Am J Obstet Gynecol 164-1771,1998. Keit,L,G; Berger,G,S:On the causation of pelvic inflammatory disease. Am J Obstet Gynecol 149-215,2002. Muller,B,R;Allen,J,et al.Pelvic Inflamatory disease after histerosalpingography.Brit J Obstet Gynecol,91-1181,1999. Toth,A,O Leary,W,M: Evidence of microbial transfer by espermatozoo.Am J O bstet Gynecol 59-556,2003. Varela,R,et col:Abceso Tuboovrico,Acta mdica Portuguesa ,p:537-542,Vol. 8,2001. Sopper,D,E:Pelvic Inflamatory disease.Infections disease.Clin of North America .831-840,vol 8;n 4.Dec 2003.

57.- Mujer de 19 aos con menarca a los 12 aos ritmo menstrual 45x4. Refiere vida sexual activa desde los 17 aos con frecuencia de 4 veces por semana. A la exploracin se encuentra acn intenso en la frente, mejillas y mentn. Acude a consulta por que desea adoptar un tratamiento anticonceptivo por va oral, lo mas adecuado es: a) Norgestimato b) Gestodeno c) Ciproterona d) Levonorgestrel El efecto antiandrognico especfico del acetato de ciproterona acta por inhibicin competitiva de la unin de la 5 - alfa - dihidrotestosterona con el receptor citoslico de las clulas blanco, que disminuye la produccin y la excrecin de sebo y el aumento y el desarrollo del vello. Es un derivado de la 17 - alfa - hidroxiprogesterona que posee accin progestgena. Su accin antigonadotrfica se suma a la del etinilestradiol. El acetato de ciproterona no posee accin estrognica sino un efecto antiestrognico, y tampoco posee accin nociva sobre la funcin de la corteza suprarrenal;

Indicaciones en la mujer: Manifestaciones de androgenizacin de grado severo, por ejemplo, hirsutismo grave, alopecia androgentica de tipo grave, a menudo acompaados por manifestaciones graves de acn y/o seborrea. Indicaciones en el hombre: Atenuacin del impulso en las desviaciones sexuales. Tratamiento antiandrgeno del carcinoma de prstata inoperable. BIBLIOGRAFA 1. Swift S. Current opinion on the classification and definition of genital tract prolapse. Curr Opin Obstet Gynecol 2002; 14: 503-7. 2. De Caro R, Aragona F, Herms A, Guidolin D, Bizzi E, Pagano F. Morphometric analysis of the fibroadipose tissue of the female pelvis. J Urol 1998; 160: 707-13. 3. Gill E, Hurt W. Pathophysiology of pelvic organ prolapse. Clin Obstet Gynecol 1998; 25(4): 757-69. 4. DeLancey, J. Anatomic aspects of vaginal eversion after Hysterectomy. Am J Obstet Gynecol. 1992; 166(6 pt 1): 1717-24.

58.- Procedimiento de eleccin para una paciente de 51 aos que presenta prolapso uterino e incontinencia urinaria de esfuerzo:

a) b) c) d)

Histerectomia total abdominal. Histerectoma vaginal reconstructiva Colpoperineoplasta. Uterosuspensin.

El prolapso genital y su tratamiento ha sido siempre un importante captulo de la ginecologa. No existe consenso clnico del concepto que define al prolapso genital como patolgico. Cierto grado de descenso y relajacin de la pared vaginal es considerado normal en la gran mayora de las mujeres, siendo ms frecuente en mujeres de mayor edad. En la poblacin general solo un 3% presenta prolapso genital severo, entendiendo como tal al de III y IV grado. Es probable que alrededor de 3 a 6% de la poblacin femenina desarrolle un prolapso de esas caractersticas en algn momento de su vida La intervencin quirrgica por excelencia para el tratamiento quirrgico del prolapso uterino es la histerectoma vaginal. Esta intervencin se clasifica como "limpia-contaminada

INDICACIONES PARA LA HISTERECTOMA VAGINAL Prolapso uterino Hemorragia uterina disfuncional Carcinoma cervico-uterino "in situ" Miomatosis uterina poco voluminosa Hiperplasia endometrial Piometra

VENTAJAS DE LA HISTERECTOMA VAGINAL No deja cicatriz abdominal Mnimo trauma abdominal Escasa hemorragia transoperatoria Mnima manipulacion intestinal Menos dolor postoperatorio CUADRO CONTRAINDICACIONES PARA LA HISTERECTOMA VAGINAL Impericia

tero muy voluminoso Tumores ovricos Endometriosis Enfermedad plvica inflamatoria Ciruga previa en tero, trompas y ovarios Cncer de endometrio Histerectoma obsttrica

59.- Acude a consulta una mujer de 25 aos, cursando su 14 semana de gestacin, por tenesmo vesical, disuria y escalofro. Por su estado actual, cul de los siguientes antimicrobianos recomendara? a) b) c) d) Ampicilina. Metronidazol. Tetraciclina. Levofloxacina.

El Metronidazol no ha mostrado efectos txicos en humanos, pero es teratognico en modelo animal. Las tetraciclinas ocasionan coloracin anormal de los dientes, hepatotoxicidad y alteracin en el desarrollo de huesos. Las sulfas podran tener un efecto deletreo en el primer trimestre dada su actividad como antimetabolitos, y en los ltimos meses pueden favorecer kernicterus en el recin nacido si es que tiene alteraciones metablicas que favorezcan anemia hemoltica. Las quinolonas se han asociado a malformaciones seas en modelos animales, y se recomienda evitarlas si existen mejores opciones. Referencia: Kasper DL, Braunwald E, Fauci AS, Hauser SL, Longo DL, Jameson JL. Harrisons Principles of Internal Medicine. McGraw Hill. 16 Ed. 789-806 pp.

60.- La mayor parte de fstulas rectovaginales aparecen: a) Como consecuencia de trauma y/o del fracaso de la reparacin de desgarros totales del perineo al momento del parto. b) En la intervencin ginecolgica por va vaginal al lesionar el recto. c) En intervenciones ginecolgicas abdominales. d) Despus de la radiacin. Las causas de las fstulas rectovaginales incluyen en primer lugar al trauma obsttrico, la enfermedad inflamatoria intestinal, la diverticulitis, infecciones, los procesos malignos y las complicaciones de la radioterapia y la ciruga. Aunque el trauma obsttrico es la causa ms comn de las fstulas rectovaginales, el desarrollo de una fstula rectovaginal despus de un parto es relativamente infrecuente. Pero para la paciente que desarrolla una de ellas es un problema devastante. El mecanismo de la injuria es un desgarro de tercer o cuarto grado en el momento del parto, as es razonable de sospechar que el esfnter subyacente puede tener algn defecto. Se han reportado rupturas del esfnter que oscilan entre un 0.4 al 6.6% de los partos vaginales Un defecto esfinteriano permanente puede influir en la tcnica de reparo a realizar. En suma, aunque los sntomas de la fstula puedan parecer que dominan el cuadro de la paciente y dejen trastornos psicolgicos, la incontinencia fecal verdadera puede estar presente tambin. De las mujeres que han sufrido un desgarro de tercer o cuarto grado durante el parto, y han sido reparadas en el mismo momento de producido el mismo, el 10% van a experimentar una falla en dicho reparo, resultando en una variedad de defectos que involucran el anorrecto, la vagina y el perin, todo lo cual requerir una correccin quirrgica en aproximadamente dos tercios de los casos. Estos defectos adems causan a la mujer que los padece serios trastornos sociales, sexuales y emocionales. Las pacientes con fstulas rectovaginales se presentan tpicamente con el pasaje de gases o heces desde la vagina. Las infecciones urinarias recurrentes y la vaginitis con una secrecin ftida pueden ser tambin formas de presentacin. Ms del 50% de las pacientes que sufren de una fstula rectovaginal secundaria a un trauma obsttrico experimentan una significativa incontinencia fecal. Muchas pacientes, no nos olvidemos, que estn post radioterapia o las internadas en geritricos, sufren de secrecin ftida, vaginitis o infecciones urinarias a repeticin que muchas veces representan tambin la forma de presentacin de esta patologa. Khurrum Baig M et al. Simple rectovaginal fistulas. Int J Colorectal Dis 2000;15:323-327. 2 Haadem K, Ohrlander S, Lingman G. Long-term ailments due to anal sphincter rupture caused by delivery: a hidden problem. Eur J Obstet Gynecol Reprod Biol 1988;27:2732. 3 Sultan AH, Kamm MA, Hudson CN, Thomas JM, Bartram CI. Anal sphincter disruption during vaginal delivery. N Engl J Med 1993;329:190511. 4 Walsh CJ, Mooney EF, Upton GJ, Motson RW. Incidence of third degree perineal tears in labour and outcome after primary repair. Br J Surg 1996;83:21821

6 1.-A woman presents with painless vaginal bleeding at 37 weeks gestation. The fetal heart rate is stable in the 150 pbm. Which of the following is indicated? a. b. c. d. e. Ultrasound examination. Nonstress test. Induction of labor. Digital examination of the cervix. Rupture of membrane.

62.- Cul es el tratamiento de eleccin para la escarlatina?: a) b) c) d) Penicilina benzatnica. Cefuroxime. Amoxicilina. Eritromicina.

Streptococcus pyogenes ( -hemoltico del grupo A) es el agente etiolgico de la escarlatina, no ofrece resistencia a la penicilina benzatnica y con dosis nica ofrece un buen ndice de curacin, erradicacin y reduccin de complicaciones. Gonzlez-Saldaa N, Infectologa Clnica Peditrica, 7 edicin, pginas 403-407.

63.- Qu rgano forma parte del sistema linfoide primario? a) Placas de Peyer. b) Timo. c) Bazo. d) Ganglios linfticos .

El timo es un rgano linfoide central que se origina del mesodermo. Este revestido por una capsula de tejido conectivo. Esta capsula penetra el parnquima tmico y lo divide en lbulos y lobulillos. La corteza representa la barrera hematotimica; es por ellos que se observa de color rojizo: por los capilares tipo I que se hallan en ella. Esta barrera no permite la entrada de linfocitos B al timo, lo que quiere decir que en este rgano solo hay linfocitos t. El timo se encuentra durante la niez, desarrolla durante la adolescencia en involuciona en edades avanzadas; su funcin es participar en la maduracin de los linfocitos T.

Referencia: RUTHERFORD R, Fifth edition, W.B Saunders Company Management of lymphatic disorders, seccin XX, 2000.

64.- El lupus eritematoso generalizado es ms frecuente en:

a) b) c) d)

La adolescencia Mujeres en edad reproductiva La infancia y personas mayores de 60 aos Personas mayores de 60 aos

EPIDEMIOLOGA El LES es una enfermedad de distribucin mundial, afecta a todas las razas aunque es de mayor gravedad en la raza negra, tiene predominio por el sexo femenino en la proporcin de 9:1, se manifiesta en cualquier edad siendo ms frecuente en la etapa productiva y reproductiva de la vida (entre 20 y 40 aos). Referencia: GUTIRREZ C, MOZO L. Diagnstico inmunolgico: enfermedades auto inmunes. En: Inmunologa Clnica. Bases moleculares y celulares. J. Pea, ed. Madrid. Arn. 107-118, 1996.

65.- La equimosis periorbitaria en paciente con trauma facial, traduce fractura de: a) b) c) d) Piso posterior. Piso medio. Piso anterior. Macizo facial.

En la valoracin ocular se deben tener en cuenta las heridas de los tejidos blandos de prpado, crnea y conjuntiva. El signo de mapache (equimosis periorbitaria bilateral) se encuentra frecuentemente asociado a las fracturas de la base anterior del crneo. (Figura 3).

Figura 3. Equimosis periorbitaria luego de trauma facial contundente.

En las heridas del prpado la localizacin es de vital importancia y es necesario anotar cuidadosamente el sitio, si se compromete el borde libre o si se lesiona el lugar de paso de la va lagrimal. El tono ocular ayudar a definir si hubo herida abierta del ojo. La presencia de enoftalmo (hundimiento del globo ocular) alertar sobre la posibilidad de una herida abierta del ojo o una fractura de las paredes orbitarias. El hipoftalmos (descenso del ojo en el eje vertical) puede estar relacionado con fractura del piso de la rbita o con fractura en el sitio donde se insertan los ligamentos suspensorios del globo ocular (tubrculo de Whitnall, ligamento de Lookwood). Referencia: 1. H. Ric Harnsberger. 2004. ISBN 848174753x Translation of PocketRadiologist - Head & Neck: Top 100 Diagnoses.

66.- El estudio de eleccin para hacer diagnstico de esofagitis es: a) b) c) d) La serie Gastro Duodenal. Ph metria. Endoscopia. Impedanciometria endolumin.

Las tcnicas de cromo endoscopia se han postulado como alternativas para disminuir los errores diagnsticos. La doctora Irene Canto, en un estudio realizado (14) en 26 pacientes con histologa de esfago de Barrett, mostr para la tincin de azul de metileno una sensibilidad del 95% y una especificidad del 97%, con valores predictivos positivo y negativo del 98 y 92%, respectivamente. Por el contrario, nuestro estudio, que incluy una muestra de 105 pacientes enrolados al azar, mostr que la sensibilidad fue de 55% y la especificidad fue del 63,6%, con valores predictivos positivo y negativo del 23 y 87%, respectivamente. Esto parecera desvirtuar el valor de esta tcnica en el contexto del trabajo cotidiano de una unidad de endoscopia, por el alto nmero de falsos positivos, y an de falsos negativos, con datos en la literatura que demuestran que hay focos de metaplasia an en la mucosa de aspecto normal

Bibliografa: Urgencias en Pediatra, Interamericana.McGraw Hill. Captulo: Urgencias Mdico Quirrgicas, Seccin XXIII, Pg. 182-194 Enciclopedia Mdico Quirrgica-Pediatra 4-014-L-10-2005.

67.- Cursa con PANCA (Ac anti-citoplasma de neutrfilos anti-mieloperoxidasa): a).- Poliarteritis nodosa clsica. b).-Poliarteritis nodosa microscpica. c).- Alveolitis alrgica extrnseca. d).- Granulomatosis de Wegener. Datos de laboratorio: El aumento de los reactantes de fase aguda suele ser menos intenso que en la PAN. (Poliarteritis Nodosa)Puede haber anemia moderada o intensa en el caso de existir hemorragia pulmonar o digestiva. Lo ms caracterstico de la PA (Poliasrteritis Nodosa ). Es la presencia de anticuerpos anticitoplasma de los neutrfilos (ANCA) con patrn perifrico en inmunosfluorescencia (p-ANCA) y que por ELISA (enzyme-linked immunosorbent assay) corresponden A anticuerpos dirigidos frente a la mieloperoxidasa (anti-MPO) 11,12. Referencia:

2. Petty R, Cassidy JT, Polyarteritis nodosa and related vasculit.

68.- Dada su cobertura cul de los siguientes antibiticos elegira para el tratamiento emprico de un absceso pulmonar en un paciente que probablemente haya presentado broncoaspiracin? a) b) c) d) Clindamicina. Isoniazida. Amikacina. Claritromicina.

La mayor parte de los abscesos en esta condicin estn ocasionados por cocos gram positivos anaerobios, para los cuales la Clindamicina tiene muy buena cobertura, adems de que sus propiedades farmacolgicas le confieren importante penetracin en la zona del absceso.

Referencia: Kasper DL, Braunwald E, Fauci AS, Hauser SL, Longo DL, Jameson JL. Harrisons Principles of Internal Medicine. McGraw Hill. 16 Ed. 1536 p.

69.- El siguiente antgeno del complejo principal de histocompatibilidad se asocia con la predisposicin a padecer artritis reumatoide.

a) HLA-B27. b) HLA-DRw52. c) HLA-B8. d) HLA-DR4.

Los aspectos genticos son determinantes en la artritis reumatoide, tanto en el inicio como en la perpetuacin de la enfermedad. Por ejemplo, se ha observado en personas de raza blanca una asociacin con varios subtipos del antgeno mayor de histocompatibilidad HDLDR4. Referencia: 1.-Fox RI, Herrmann ML, Frangou CG, et al. Mechanism of action for leflunomide in rheumatoid arthritis. Clin Immunol 1999; 93(3): 198-208.

70.- El estadio 2 de la nefropata diabtica se caracteriza por aparicin de:

a) b) c) d)

Hipertensin arterial sistmica. Microalbuminuria. Eritrocituria. Proteinuria.

Clasificacin de nefropata diabtica.

NEFROPATIA DIABETICA
Estadio I: Hiperfiltracin. Estadio II: Microalbuminuria (30 a 300 mg/d). 6 a 15 aos. Estadio III: Proteinuria. Hipertensin. Perdida de GFR 10 ml/min al ao. Estadio IV: Insuficiencia renal.

Uso exclusivo Universidad La Salle , Dr. Cano

Referencia:

1.Report of the Expert Committee on the Diagnosis and Classification of Diabetes Mellitus. Diabetes Care. 1997;20:1183-97.

71.- El agente causal ms frecuente de la otitis media aguda es? a) b) c) d) Haemophilus influenza tipo B. Streptococcus pneumoniae. Staphylococcus aureus. Moraxella catarrhalis.

El Streptococcus pneumoniae representa del 30 al 35 % de los casos de otitis media aguda desde las revisiones de Feigin (1981), Ruuskanen y Heikkinen (1994) y Blumer (1997). Gonzlez-Saldaa N, Infectologa Clnica Peditrica, 7 edicin, pginas 63-98.

72.- La maniobra para efectuar la reduccin de la subluxacin de cabeza de radio en nios debe ser en:

a) Extensin y pronacin de codo. b) Flexin y pronacin de codo. c) Extensin y supinacin de codo. d) Flexin y supinacin de codo.

Subluxacin de la cabeza del radio fuera del ligamento anular, que ocurre en nios de entre uno y tres aos como resultado de una traccin axial sobre el miembro superior Cursa con dolor, codo en semiextensin, antebrazo en pronacin y ausencia de movilidad activa de la extremidad superior afecta. El tratamiento consiste en supinar forzadamente el antebrazo en extensin y a continuacin flexionarlo. Manual CTO 7 edicin, Traumatologa y Ortopedia, Pg. 37

73.- De los siguientes antibiticos, en cul es indispensable ajustar la dosis de acuerdo a la funcin renal? a) Anfotericina B. b) Dicloxacilina. c) Metronidazol. d) Ofloxacina.

Aun cuando algunos de los antibiticos mencionados tienen nefrotoxicidad (Anfotericina) o pertenecen a grupos en los que otros compuestos requieren de ajuste a la funcin renal, como los betalactmicos (dicloxacilina), en este grupo slo la ofloxacina, parte de las quinolonas, requiere de ajuste para la funcin renal. Referencia:

Kasper DL, Braunwald E, Fauci AS, Hauser SL, Longo DL, Jameson JL. Harrisons Principles of Internal Medicine. McGraw Hill. 16 Ed. 789-806 pp.

74.- El signo clnico para diagnostico de escoliosis se conoce como: a) Signo de Cajn. b) Signo de Allis. c) Signo de McMurray. d) Signo de Adams. Esta rotacin se detecta clnicamente con el test de Adams, cuando el paciente flexiona su columna, existe asimetra de la posicin de la parrilla costal y/o los flancos lumbares. Referencia: Skinner, H. Diagnstico y tratamiento en Ortopedia. Ed. Manual Moderno. Mxico, 2004. pp. 256.

75.- La metas de control glucmico en el paciente diabtico adulto son: a) b) c) d) Glucosa de ayuno < 180, 2 horas postprandial < 240, HbA1c <8 Glucosa de ayuno: 140-180, 2 horas postprandial < 200, HbA1c <7 Glucosa de ayuno: 70-100, 2 horas postprandial < 140, HbA1c < 6 Glucosa de ayuno: 90-130, 2 horas postprandial < 180, HbA1c < 7

Para definir con claridad los objetivos del tratamiento de la hiperglucemia, los expertos recomiendan alcanzar y mantener rangos no diabticos, es decir, una glucemia en ayunas inferior a 130 mg/dL, junto con glucemias postprandiales menores de 180 mg/dL (idealmente menores a 140 mg/dL) y cifras de hemoglobina glucosilada (HbA1c) menores a 7%. Referencia: Report of the Expert Committee on the Diagnosis and Classification of Diabetes Mellitus. Diabetes Care. 1997;20:1183-97. 2. Sheperd PR, Kahn BB. Mechanisms of disease: Glucose transporters and insulin action. Implications for insulin resistance and diabetes mellitus. N Engl J Med, 1999;341:248-257. 3. Bloomgarden ZT. Obesity and diabetes. Diabetes Care. 2000;23:1584-1590. Belfiore F, Lannello S, Camuto M, Fagone S, Calaveri A. Insulin sensitivity of blood glucose versus insulin sensitivity of blood free acids in normal, obese, and obesediabetic subjects. Metabolism. 2001;50:573-582. 1.

76.- Factores que favorecen la hidronefrosis:

a) b) c) d)

Pelvis intrarrenal Cicatrices de pelvis Pelvis pequea Pelvis extrarenal

La hidronefrosis bilateral se desarrolla cuando la pelvis y los clices (estructuras que recolectan la orina de los riones) de ambos riones se dilatan, debido a que la orina no puede fluir del rin hacia la vejiga a travs de los urteres (bilateral significa ambos lados).

Referencia:
1. Review Date: 9/13/2005. 2. Reviewed by: Robert Mushnick, M.D., Clinical Assistant Professor, SUNY Downstate Health Center, Brooklyn, NY. Review provided by VeriMed Healthcare Network.

77.- En una fstula gstrica el principal electrolito que se pierde es? a) Na b) K c) Cl d) Ca

Las secreciones gstricas por da son: 1000-2000ml, 60-90 meq/L Na, 10-30 meq/L K, 100-130 meq/L Cl y 0 meq/L HCO3, constituyendo el CL el principal electrolito de la secrecin gstrica. Referencia: Brunicardi, et al. Schwartz, Principios de Ciruga. Octava Edicin, Edit. McGraw Hill, pag 4.

78.- Menciona cul es la cardiopata ms frecuente en el sndrome de Down?

a) b) c) d)

La transposicin de grandes vasos. La coartacin artica. Defecto atrioventricular. La comunicacin interauricular.

De los 275 nios estudiados, y que cursaron con cardiopata 160 (58%). Las cardiopatas que se presentaron con mayor frecuencia fueron la comunicacin interauricular (CIA), comunicacin interventricular (CIV) y persistencia del ductus arterioso (PDA) (90%); nicamente 14 casos (9%) correspondieron a defectos de la tabicacin auriculoventricular, a diferencia de lo observado en otros pases. La manifestacin clnica ms frecuente fue la insuficiencia cardaca. El 15% de los pacientes (n = 25) fallecieron, y las causas ms frecuentes fueron el choque sptico y cardiognico. De los pacientes con cardiopata congnita, el 4-10% se asocia a SD, y el 40-60% de los pacientes con este sndrome presenta cardiopata congnita. La malformacin cardiaca es la mayor causa de mortalidad en los primeros 2 aos de la vida , . En Mxico, las cardiopatas que se presentan con ms frecuencia en los nios con SD son la persistencia del ductus arterioso (PDA), la comunicacin interventricular (CIV) y la comunicacin interauricular (CIA), a diferencia de lo mencionado en pases anglosajones y europeos, donde los defectos de la tabicacin atrioventricular (D-AV) son los ms comunes (40-70%) y, entre estos, el canal atrioventricular parcial (CIA ostium primum con hendidura mitral) es el ms frecuente
5-10 5

Referencia: 56,mero 0 1. Maroto CM, Enrquez de Salamanca F, Herriz IS, Zabala JA.. Guas de prctica clnica de la Sociedad Espaola de Cardiologa en las cardiopatas congnitas ms frecuentes. Rev Esp Cardiol 2001;54:67-82.

79.- El electrolito ms abundante en la saliva es? a) Na b) K c) Cl d) Ca

La saliva total es un complejo de secretados multiglandulares que tienen unas fluctuaciones cualitativas muy superiores a las que suele tener la lgrima. Las caractersticas bioqumicas y biofsicas de la saliva total no se separan mucho de las de la lgrimas, y en aquellos

parmetros en que ms se separan, la experiencia ha mostrado que son bien tolerados por la superficie ocular. Lgrima Peso especfico Tensin superficial Presin osmtica pH Viscosidad a 37C 1004-1008 40 dinas/cm 300-310 mOsm/l 7'47 1 centipoise Saliva total 1002-1012 15-26 dinas/cm 200-310 mOsm/l 6'75-7'25 2-3 centipoises

Los componentes orgnicos de la saliva son seroalbmina, seroglobulina, urea, cido rico, creatinina, mucina y algunos aminocidos y enzimas tales como triptfano, amilasa salival (ptialina) y lisozima. Los componentes inorgnicos ms importantes son cloruro sdico, cloruro potsico, bicarbonato sdico, fosfatos monosdico y disdico, fosfato clcico y sulfocianato potsico. Desde milenios atrs se sabe que el ojo tolera satisfactoriamente la saliva. TRATADO DE FISIOLOGA MDICA Arthur C. Guyton (Editorial McGraw-Hill). Ed. 2006. Cap. Liquidos, electrolitos.

80.- Cul es el lugar de produccin del humor acuoso? a) b) c) d) Trabcula. Procesos ciliares. Pars plana. Glndulas lagrimales accesorias.

El cuerpo ciliar esta formado por el msculo ciliar y la porcin epitelial compuesta por pars plana y pars plicata o procesos ciliares, responsables de la produccin del humor acuoso.

Das könnte Ihnen auch gefallen